Sie sind auf Seite 1von 88

Sports Medicine

Self-Assessment Examination 2013

AnSwER BooK
CME for this program expires
on December 31, 2015
All rights reserved. No part of Sports Medicine Self-Assessment Examination may be reproduced, stored
in a retrieval system, or transmitted in any form or by any means (electronic, mechanical, photocopying,
recording, or otherwise) without the prior written permission of the publisher.

Published January 2013


American Academy of Orthopaedic Surgeons
6300 North River Road
Rosemont, IL 60018

Copyright 2013 by the American Academy of Orthopaedic Surgeons

Requests for permission to reproduce any part of the work should be mailed to:
Attention: Examinations Department
American Academy of Orthopaedic Surgeons
6300 North River Road
Rosemont, IL 60018

Printed in the USA

2013 American Academy of Orthopaedic Surgeons 2013 Sports Medicine Self-Assessment Examination

2013 American Academy of Orthopaedic Surgeons 2013 Sports Medicine Self-Assessment Examination
Dear Colleague:

Thank you for purchasing the AAOS 2013 Sports Medicine Special Interest Examination. This
exam has been designed to reflect current diagnostic and therapeutic scenarios that sports
medicine orthopaedic physicians will encounter in their daily practice. Several emerging
domains represent new areas of focus in this years exam, while some traditional topics receive
less attention.

The target audience for this examination is practicing sports medicine orthopaedic physicians.
Historically, purchasers of this exam are preparing for Maintenance of Certification (MOC) or
sports medicine subspecialty certification (CAQ), or are taking the exam to further and update
their knowledge of current sports medicine issues. Our specialty continues to evolve every year,
with an improved understanding of injury, treatment, and prevention. The content of this
examination is designed to reflect these advances and our ambition to provide the best care
possible to our patients.

We are increasingly tasked with recognizing and treating our patients many medical
conditions. This examinations questions have been crafted to challenge the examinees
knowledge, interpretation of diagnostic imaging, and problem-solving skills. The question
format has been designed to test each of these skill sets. My goal was to create clinically
relevant scenarios that represent the common diagnostic and treatment challenges that each
of us must face. Areas of focus include femoroacetabular impingement, advances in anterior
cruciate ligament reconstruction, management of chondrosis and arthrosis, and issues of
concern for adolescent athletes. Concussion management and methicillin-resistant
Staphylococcus Aureus infections also are areas of particular concentration in this examination.

Thank you for selecting the 2013 Sports Medicine Special Interest Examination. The Sports
Medicine Evaluation Committee, the AAOS examination staff, and I hope you will regard this
exam as a valuable educational resource.

Sincerely,

James C. Dreese, MD
Editor in Chief, 2013 Sports Medicine Examination

2013 American Academy of Orthopaedic Surgeons 2013 Sports Medicine Self-Assessment Examination
2013 American Academy of Orthopaedic Surgeons 2013 Sports Medicine Self-Assessment Examination
ACCREDITATION
The American Academy of Orthopaedic Surgeons is accredited by the Accreditation Council for
Continuing Medical Education to provide continuing medical education for physicians.

AMA PRA CREDIT


U. S. Physicians: The American Academy of Orthopaedic Surgeons designates this enduring
material for a maximum of 10 AMA PRA Category 1 Credits. Physicians should claim only
the credit commensurate with the extent of their participation in the activity.

International Physicians: AMA PRA credit may only be claimed by, and awarded to,
physicians, defined by the AMA as individuals who have completed an allopathic (MD),
osteopathic (DO), or an equivalent medical degree from another country.

Allied Health Professionals: The American Academy of Orthopaedic Surgeons is not accredited to
offer credit for nurses and other allied health professionals. To determine if activities offering AMA PRA
Category 1 Credits are acceptable for your licensing or certification needs, please contact the relevant
organization(s) directly.

Independent Study (previously called Self-Scored) Examination Participants: The AAOS no


longer awards CME credit for printed independent study self-assessment examinations as a result
of new requirements from the AMAs Physician Recognition Award Program.

EDUCATIONAL OBJECTIVES
As a result of taking the 2013 Sports Medicine Self-Assessment Examination, I
will update my concussion management in accordance with accepted international guidelines.
can promptly diagnose and treat common athletic soft tissue disorders.
can discuss with my patients the available treatment options for osteoarthritis.
can counsel families regarding common adolescent athletic injuries and effective
prevention/treatment strategies.
expanded my knowledge of femoroacetabular impingement and effective treatment options.
expanded my understanding of the principles of anatomic ACL reconstruction.

INQUIRIES
Questions regarding completing this CME activity or other comments may be sent to
exams@aaos.org or write to: Attention: Examinations Department, American Academy of
Orthopaedic Surgeons, 6300 North River Road, Rosemont, IL 60018.

RELEASE DATE: January 2013


EXPIRATION DATE: December 31, 2015.
No CME credit will be awarded for this activity after December 31, 2015

2013 American Academy of Orthopaedic Surgeons 2013 Sports Medicine Self-Assessment Examination

2013 American Academy of Orthopaedic Surgeons 2013 Sports Medicine Self-Assessment Examination
Produced by the American Academy of Orthopaedic Surgeons
Sports Medicine Evaluation Committee of the Central Evaluation Committee

James C. Dreese, MD, Editor in Chief


Assistant Professor, Orthopaedic Surgery, University of Maryland School of Medicine,
Baltimore, Maryland

Contributors
Gregory F. Carolan, MD
Clinical Assistant Professor, Department of Orthopaedic Surgery, Temple University School
of Medicine, Philadelphia, Pennsylvania; Orthopaedic Sports Medicine Section Chief,
Department of Orthopaedic Surgery, St. Luke's University Hospital, Bethlehem,
Pennsylvania
C. David Geier Jr, MD
Assistant Professor of Orthopaedic Surgery, Director MUSC Sports Medicine, Medical
University of South Carolina, Charleston, South Carolina
Brett W. Gibson, MD
Clinical Assistant Professor, Department of Orthopaedic Surgery, Temple University School
of Medicine, Philadelphia, Pennsylvania; Orthopaedic Surgeon, St. Luke's Orthopaedic
Specialists, Bethlehem, Pennsylvania
Laith M. Jazrawi, MD
Associate Professor of Orthopaedic Surgery, Chief of the Division of Sports Medicine at
New York University Hospital for Joint Diseases, New York, New York
Navin R. Kilambi, MD
Assistant Clinical Professor, University of Louisville Medical School, Louisville, Kentucky;
Orthopaedic Surgeon, Private Practice, Orthopaedic Associates, Jewish Physician's Group,
Louisville, Kentucky
Faisal Mirza, MD, FRCSC
Orthopaedic Surgeon, Medical Officer, MDFP Fellow, Orthopaedic Device Evaluation
Regulation & Research, Food and Drug Administration, Silver Spring, Maryland
Luke Oh, MD
Attending Orthopaedic Surgeon, Massachusetts General Hospital; Instructor in
Orthopaedic Surgery, Harvard Medical School, Boston, Massachusetts
Albert W. Pearsall IV, MD
Professor, Director, Section of Sports Medicine, Department of Orthopaedic Surgery,
University of South Alabama, Mobile, Alabama
Daniel J. Solomon, MD
Private Practice, Marin Orthopedics and Sports Medicine, Novato, California; Assistant
Professor, Uniformed Services University of Health Sciences, Bethesda, Maryland
Fotios P. Tjoumakaris, MD
Assistant Professor of Orthopaedic Surgery, Jefferson Medical College, Thomas Jefferson
University, Philadelphia, Pennsylvania; Orthopaedic Surgeon, Rothman Institute, Egg
Harbor Township, New Jersey

2013 American Academy of Orthopaedic Surgeons 2013 Sports Medicine Self-Assessment Examination

2013 American Academy of Orthopaedic Surgeons 2013 Sports Medicine Self-Assessment Examination
ACKNOWLEDGMENTS

Medical Editing
Brenda Moss Feinberg, ELS, AAOS Examinations Consultant

Graphic Design and Digital Media Specialist


John Cisco, AAOS Examinations Consultant

American Academy of Orthopaedic Surgeons


Constance M. Filling, Chief Education Officer

Department of Electronic Media, Examinations, CME Course Operations, and Practice Management
Howard Mevis, Director
Laura Hruska, M. Ed., Manager, Examinations
Marcie L. Lampert, Senior Coordinator, Examinations
Anna M. Scheer, Senior Coordinator, Examinations
Denise Plasky, Assistant, Examinations
Irene Bogdal, Administrative Assistant, Examinations

2013 American Academy of Orthopaedic Surgeons 2013 Sports Medicine Self-Assessment Examination

2013 American Academy of Orthopaedic Surgeons 2013 Sports Medicine Self-Assessment Examination
Disclosure Information for the 2013 Sports Medicine Self-Assessment Examination:

James C. Dreese, MD (Editor in Chief): 2 (Cayenne Medical);3B (Cayenne Medical); Submitted on:
06/01/2012. *
Gregory F. Carolan, MD (Member): 2 (Mitek);3B (Mitek);8 (Arthroscopy); Submitted on:
06/03/2012. *
C. David Geier, Jr MD (Member): 9 (AAOS; American Orthopaedic Society for Sports Medicine,
South Carolina Medical Association); Submitted on: 04/01/2012. *
Brett W. Gibson, MD (Member): (n) Submitted on: 04/16/2012. *
Laith M. Jazrawi, MD (Member): 3B (Ferring Pharmaceuticals, CONMED Linvatec, Knee
Creations, CORE Essence, Depuy Mitek); Submitted on: 04/02/2012. *
Navin R. Kilambi, MD (Member): 9 (AAOS - Sportsmedicine Evaluation Sub-Committee);
Submitted on: 03/25/2012. *
Faisal Mirza, MD, FRCSC (Member): 9 (AAOS Sports Medicine Evaluation Subcommittee;
AAOS); Submitted on: 04/03/2012. *
Luke S. Oh, MD (Member): (n) Submitted on: 04/03/2012. *
Albert W. Pearsall IV, MD (Member): 3B (Biomet);5 (LUITPOLD PHARMACEUTICALS,
INC);9 (LifeLink); Submitted on: 06/01/2012. *
Daniel J. Solomon, MD (Member): 2 (Arthrex, Inc; Pacific Medical);8 (Arthroscopy; American
Orthopedic Sports Medicine Society Sports Medicine Update);9 (AAOS; American Orthopaedic
Society for Sports Medicine); Submitted on: 06/01/2012. *
Fotios P. Tjoumakaris, MD (Member): 2 (Ferring Pharmaceuticals); Submitted on: 05/10/2012. *

American Academy of Orthopaedic Surgeons Staff


Howard Mevis (Rosemont, IL): 4 (GE Healthcare; 3M; Novartis); 9 (Orthopaedic Learning Center);
Submitted on: 09/05/2012. *
Laura Hruska (Rosemont, IL): (n) Submitted on: 04/30/2012. *
Marcie Lampert (Staff Liaison): (n) Submitted on: 09/07/2012. *

* Disclosure Items Answered: (n) = Respondent answered 'No' to all items indicating no conflicts.
1= Royalties from a company or supplier; 2= Speakers bureau/paid presentations for a company or supplier; 3A=
Paid employee for a company or supplier; 3B= Paid consultant for a company or supplier; 3C= Unpaid consultant
for a company or supplier; 4= Stock or stock options in a company or supplier; 5= Research support from a company
or supplier as a PI; 6= Other financial or material support from a company or supplier; 7= Royalties, financial or
material support from publishers; 8= Medical/Orthopaedic publications editorial/governing board; 9= Board
member/committee appointments for a society.

2013 American Academy of Orthopaedic Surgeons 2013 Sports Medicine Self-Assessment Examination
Disclaimer
The material presented in this self-assessment examination has been made available by the
American Academy of Orthopaedic Surgeons for educational purposes only. This material is not
intended to present the only, or necessarily the best, methods or procedures for the medical
situations discussed, but rather is intended to represent an approach, view, statement, or opinion
of the author(s) or producer(s), that may be helpful to others who face similar situations.

Some drugs or medical devices demonstrated in Academy courses or described in Academy print
or electronic publications have not been cleared by the Food and Drug Administration (FDA) or
have been cleared for specific uses only. The FDA has stated that it is the responsibility of the
physician to determine the FDA clearance status of each drug or device he or she wishes to use in
clinical practice.

Furthermore, any statements about commercial products are solely the opinion(s) of the author(s)
and do not represent an Academy endorsement or evaluation of these products. These statements
may not be used in advertising or for any commercial purpose.

2013 American Academy of Orthopaedic Surgeons 2013 Sports Medicine Self-Assessment Examination
2013 Sports Medicine Self-Assessment Examination Answer Book 11

Figure 1

Question 1
Figure 1 is the MRI scan of a 19-year-old man who has an acute anterior shoulder dislocation. The bony
fragment occupies 10% of the glenoid articular surface. What is the most appropriate treatment?

1. Open structural iliac crest graft


2. Open reduction and internal fixation
3. Arthroscopic coracoid transfer
4. Arthroscopic repair incorporating the bone lesion

PREFERRED RESPONSE: 4

DISCUSSION
The MRI scan shows a bony Bankart lesion involving less than 20% of the glenoid joint surface. A
recent series reported high success rates after arthroscopic treatment when the defect is incorporated
into the repair. Anterior bony deficiencies occupying more than 25% to 30% of the glenoid joint surface
treated with soft-tissue repair only are associated with high recurrence rates. In these patients, an open
or arthroscopic coracoid transfer or structural iliac crest graft should be considered. Open reduction
and internal fixation has been reported for treatment of large acute glenoid rim fractures but is not
recommended for recurrent anterior shoulder instability in the setting of a 10% glenoid rim fracture.

RECOMMENDED READINGS
Getz CL, Buzzell JE, Krishnan SG. Shoulder instability and rotator cuff tears. In: Flynn JM, ed.
Orthopaedic Knowledge Update 10. Rosemont, IL: American Academy of Orthopaedic Surgeons;
2011;299-314.
Sugaya H, Moriishi J, Kanisawa I, Tsuchiya A. Arthroscopic osseous Bankart repair for chronic recurrent
traumatic anterior glenohumeral instability. J Bone Joint Surg Am. 2005 Aug;87(8):1752-60. PubMed
PMID: 16085615.

2013 American Academy of Orthopaedic Surgeons 2013 Sports Medicine Self-Assessment Examination
12 American Academy of Orthopaedic Surgeons

Question 2
A 19-year-old running back lands directly on his anterior knee after being tackled. He has mild anterior
knee pain, a trace effusion, a 2+ posterior drawer, a grade 1+ stable Lachman, no valgus laxity, and
negative dial tests at 30 degrees and 90 degrees. What is the best treatment strategy at this time?

1. Physical therapy with a focus on quadriceps strengthening


2. Physical therapy and delayed posterior cruciate ligament (PCL) reconstruction
3. PCL reconstruction
4. PCL and posterolateral corner reconstruction

PREFERRED RESPONSE: 1

DISCUSSION
This patient has likely sustained an isolated PCL injury. The examination is consistent with a grade II
injury to the PCL. In this scenario, the best initial option is nonsurgical treatment and return to play as
symptoms subside and strength improves. Physical therapy with a focus on quadriceps strengthening and
delayed PCL reconstruction is not the answer because this patient can likely be treated without surgery.
The absence of valgus laxity and negative dial testing findings suggest that an injury to the posteromedial
and posterolateral corners has not occurred. Initial nonsurgical treatment is indicated for this patient. If he
completes rehabilitation and experiences persistent disability with anterior and/or medial knee discomfort
or senses the knee is loose, PCL reconstruction should be considered at that time.

RECOMMENDED READINGS
Shelbourne KD, Davis TJ, Patel DV. The natural history of acute, isolated, nonoperatively treated
posterior cruciate ligament injuries. A prospective study. Am J Sports Med. 1999 May-Jun;27(3):276-83.
PubMed PMID: 10352760.
McAllister DR, Petrigliano FA. Diagnosis and treatment of posterior cruciate ligament injuries. Curr
Sports Med Rep. 2007 Oct;6(5):293-9. Review. PubMed PMID: 17883964.

2013 American Academy of Orthopaedic Surgeons 2013 Sports Medicine Self-Assessment Examination
2013 Sports Medicine Self-Assessment Examination Answer Book 13

Figure 3

Question 3
Figure 3 is the clinical photograph of a 20-year-old college soccer player who has a 7-day history of
worsening left ankle pain and swelling after being slide-tackled in a game. Radiograph findings of his
ankle and foot are normal. He complains of malaise. His history includes a severe ankle sprain 3 months
ago. The sprain caused him to miss half the season, but he was able to play in the last 2 games. What is
the most appropriate treatment?

1. Incision and drainage


2. Ice the ankle but don't let him play.
3. Topical antibiotics for 7 days with an occlusive dressing
4. Debridement in the training room followed by 5 days of oral antibiotics

PREFERRED RESPONSE: 1

DISCUSSION
The clinical photograph shows a skin infection with an appearance consistent with methicillin-resistant
Staphylococcus aureus. This infection should be clinically incised and allowed to drain and a course of
antibiotics should follow. If this infection is not promptly treated with debridement, it likely will worsen
and potentially spread to other teammates. Antibiotics are secondary to surgical debridement but are a
necessary adjunct. Although this patient has a history of severe sprain, his malaise and skin appearance do
not correlate with a ligament injury or fracture. Debridement in the training room is not appropriate and
would likely not fully decompress the fluid collection.

RECOMMENDED READINGS
Kirkland EB, Adams BB. Methicillin-resistant Staphylococcus aureus and athletes. J Am Acad Dermatol.
2008 Sep;59(3):494-502. Epub 2008 Jun 11. Review. PubMed PMID: 18550208.
Rihn JA, Michaels MG, Harner CD. Community-acquired methicillin-resistant staphylococcus aureus:
an emerging problem in the athletic population. Am J Sports Med. 2005 Dec;33(12):1924-9. Review.
PubMed PMID: 16314668.

2013 American Academy of Orthopaedic Surgeons 2013 Sports Medicine Self-Assessment Examination
14 American Academy of Orthopaedic Surgeons

RESPONSES FOR QUESTIONS 4 AND 5


1. Semimembranosis tendonitis
2. Patellar tendonitis
3. Iliotibial band friction syndrome
4. Quadriceps tendonitis

Please select the most likely diagnosis listed above for each clinical situation.

Question 4
A 23-year-old otherwise healthy 6-ft, 4-in basketball player complains of pain in his knees. An
examination reveals localized tenderness to palpation over the inferior pole of the patella. The patient
notes a significant exacerbation of his pain when the examiner takes the knee from flexion to extension.

PREFERRED RESPONSE: 2

Question 5
A 22-year-old 6-ft, 2-in Olympic cyclist has had knee pain for 2 months. Examination reveals localized
tenderness to palpation over the lateral femoral epicondyle most notably at 30 degrees of flexion.

PREFERRED RESPONSE: 3

DISCUSSION FOR QUESTIONS 4 AND 5


Patellar tendonitis is common in jumping sports such as basketball and volleyball. The pain is localized
to the inferior border of the patella and is exacerbated by extension of the knee. Treatment for the vast
majority of patients is nonsurgical and includes nonsteroidal anti-inflammatory drugs, physical therapy,
and orthoses (patella tendon strap). Iliotibial band friction most commonly occurs in cyclists and runners
(especially those who run up hills) and is a result of abrasion between the iliotibial band and the lateral
femoral condyle. Localized tenderness with the knee flexed at 30 degrees is common. The Ober test
may be helpful in making the diagnosis. Semimembranosis tendonitis most commonly occurs in male
athletes during their fourth decade of life. The diagnosis is usually made with an MRI scan or nuclear
imaging. Quadriceps tendonitis is similar to patellar tendonitis but is much less common. The pain may
be associated with clicking and is localized to the superior border of the patella.

RECOMMENDED READINGS QUESTIONS 4 AND 5


Hosea TM, Tria AJ. Physical examination of the knee: clinical. In: Scott WN, ed. Ligament and Extensor
Mechanism Injuries of the Knee: Diagnosis and Treatment. St Louis, MO: CV Mosby; 1991.
Kodali P, Islam A, Andrish J. Anterior knee pain in the young athlete: diagnosis and treatment. Sports Med
Arthrosc. 2011 Mar;19(1):27-33. Review. PubMed PMID: 21293235.

END OF SERIES

2013 American Academy of Orthopaedic Surgeons 2013 Sports Medicine Self-Assessment Examination
2013 Sports Medicine Self-Assessment Examination Answer Book 15

Question 6
A 26-year-old weightlifter had increasing pain in his left shoulder for 4 months. Nonsurgical treatment
consisting of anti-inflammatory medication, corticosteroid injections, and rest failed to alleviate his
symptoms. He underwent an arthroscopic distal clavicle resection with excision of the distal 8 mm of
clavicle (Mumford procedure). Three months after surgery, he reported popping by his clavicle and mild
pain. His clavicle demonstrated mild posterior instability on examination without any obvious deformity
on his radiographs. What structures were compromised during his excision?

1. Anterior and superior acromioclavicular joint ligaments


2. Posterior and superior acromioclavicular joint ligaments
3. Conoid ligament
4. Trapezoid ligament

PREFERRED RESPONSE: 2

DISCUSSION
The posterior and superior acromioclavicular ligaments provide the most restraint to posterior translation
of the acromioclavicular joint and must be preserved during a Mumford procedure. Anterior and
superior acromioclavicular joint ligaments are the opposite of the preferred response and prevent anterior
translation of the clavicle. Injuries to the conoid and trapezoid ligaments are more pronounced with
grade III or higher acromioclavicular separations, with superior migration of the clavicle relative to the
acromion.

RECOMMENDED READINGS
Strauss EJ, Barker JU, McGill K, Verma NN. The evaluation and management of failed distal clavicle
excision. Sports Med Arthrosc. 2010 Sep;18(3):213-9. Review. PubMed PMID: 20711054.
Nuber GW, Bowen MK. Arthroscopic treatment of acromioclavicular joint injuries and results. Clin Sports
Med. 2003 Apr;22(2):301-17. Review. PubMed PMID: 12825532.

2013 American Academy of Orthopaedic Surgeons 2013 Sports Medicine Self-Assessment Examination
16 American Academy of Orthopaedic Surgeons

CLINICAL SITUATION FOR QUESTIONS 7 THROUGH 9


A 19-year-old female field hockey player has a right ankle injury that occurred last night during a game.
The patient is on crutches and states that she has not been able to put any weight on her right ankle
since the injury. She was running alongside with another player when her right ankle gave out and
she twisted it, falling to the ground. Physical examination revealed discoloration similar to a hematoma
and significant swelling around the lateral ankle area. Pain was elicited during palpation of the anterior
talofibular ligament.

Question 7
What examination test should be performed to aid in this diagnosis?

1. Thompson test
2. External rotation stress test
3. Anterior drawer test
4. Squeeze test

PREFERRED RESPONSE: 3

Question 8
Radiographs of the players right ankle confirm there are no fractures. With a lateral talar tilt test result of
19 degrees, which additional structure is most likely damaged?

1. Deltoid ligament
2. Calcaneofibular ligament
3. Anterior tibiofibular ligament
4. Posterior tibiofibular ligament

PREFERRED RESPONSE: 2

Question 9
What is the most appropriate course of action for this patients condition?

1. Early mobilization and a guided proprioceptive and strengthening rehabilitation program


2. Extended immobilization in a cast
3. Surgical intervention
4. Weight bearing as tolerated in an ankle brace for 6 weeks

PREFERRED RESPONSE: 1

2013 American Academy of Orthopaedic Surgeons 2013 Sports Medicine Self-Assessment Examination
2013 Sports Medicine Self-Assessment Examination Answer Book 17

DISCUSSION FOR QUESTIONS 7 THROUGH 9


The anterior drawer test is performed with the ankle in 10 degrees of plantar flexion, which results in
the greatest amount of translation. The test investigates the integrity of the anterior talofibular ligament
with a key distance of translation being 8 to 10 mm. While the patient is sitting and has her knees flexed
over the edge of a table or bench, the physician or examiner uses one hand to stabilize the distal leg and
with the other applies an anterior force to the heel in an attempt to gap the talus anteriorly from under the
tibia. The anterior talofibular ligament and calcaneofibular ligament are both compromised based on the
examination findings. The anterior drawer test result reflects injury to the anterior talofibular ligament
and a possible injury to the calcaneofibular ligament. A lateral talar tilt test angle measurement greater
than 15 degrees reflects a rupture of both anterior talofibular ligament and calcaneofibular ligaments.
The diagnosis is a severe lateral ligament complex sprain. Considering the involvement of the anterior
talofibular ligament and calcaneofibular ligaments, early mobilization with a cast or controlled ankle
movement walker boot has been documented to result in better patient outcomes than compression or air
casting.

RECOMMENDED READINGS FOR QUESTIONS 7 THROUGH 9


Anderson RB, James WC III, Lee S. Athletic foot disorders. In: Garrick JG, ed. Orthopaedic Knowledge
Update: Sports Medicine 3. Rosemont, IL: American Academy of Orthopaedic Surgeons; 2004:249-261.
Osborne MD, Rizzo TD Jr. Prevention and treatment of ankle sprain in athletes. Sports Med.
2003;33(15):1145-50. PubMed PMID: 14719982.
Hertel J. Immobilisation for acute severe ankle sprain. Lancet. 2009 Feb 14;373(9663):524-6. PubMed
PMID: 19217974.
Stiell IG, Greenberg GH, McKnight RD, Nair RC, McDowell I, Reardon M, Stewart JP, Maloney J.
Decision rules for the use of radiography in acute ankle injuries. Refinement and prospective validation.
JAMA. 1993 Mar 3;269(9):1127-32. PubMed PMID: 8433468.
Tropp H, Askling C, Gillquist J. Prevention of ankle sprains. Am J Sports Med. 1985 Jul-
Aug;13(4):259-62. PubMed PMID: 3927758.

END OF SERIES

2013 American Academy of Orthopaedic Surgeons 2013 Sports Medicine Self-Assessment Examination
18 American Academy of Orthopaedic Surgeons

Question 10
A 17-year-old basketball player has a soft-tissue abscess over the anterior aspect of his left knee. The
team physician prescribes amoxicillin and the infection resolves. The next week the patient develops
fevers and significantly increased pain at the site of the previous infection. What is the most likely
diagnosis?

1. Community-acquired methicillin-resistant Staphylococcus aureus (CA-MRSA)


2. Tinea corporis
3. Herpes simplex virus
4. Group A Streptococcus

PREFERRED RESPONSE: 1

DISCUSSION
Skin and soft-tissue abscesses should be drained and cultured by the treating physician whenever possible.
Antibiotic therapy should be guided by antibiotic sensitivities derived from the cultures to identify cases
of CA-MRSA and prevent severe recurrent infections. These infections have been associated with
significant morbidity, with up to 70% of players requiring hospitalization. A high index of suspicion in
at-risk populations is necessary, and empiric treatment with an antibiotic effective against MRSA should
be considered until sensitivity results are available. Tinea corporis is a general term for a cutaneous
fungal infection. The lesion appears as a well-demarcated erythematous plaque with a raised border and
central hypopigmentation, giving it a ring-like appearance. Primary infection with herpes simplex virus
can produce constitutional symptoms with burning, tingling, or stinging at the site. Grouped vesicles with
clear fluid 1 mm to 2 mm in size form on an erythematous base and then rupture, leaving moist ulcers or
crusted plaques. Amoxicillin is appropriate empiric antibiotic therapy for group A Streptococcus, so a
recurrent infection is less likely with this organism.

RECOMMENDED READINGS
Rihn JA, Michaels MG, Harner CD. Community-acquired methicillin-resistant staphylococcus aureus:
an emerging problem in the athletic population. Am J Sports Med. 2005 Dec;33(12):1924-9. Review.
PubMed PMID: 16314668.
Marcotte AL, Trzeciak MA. Community-acquired methicillin-resistant Staphylococcus aureus: an
emerging pathogen in orthopaedics. J Am Acad Orthop Surg. 2008 Feb;16(2):98-106. Review. PubMed
PMID: 18252840.

2013 American Academy of Orthopaedic Surgeons 2013 Sports Medicine Self-Assessment Examination
2013 Sports Medicine Self-Assessment Examination Answer Book 19

Figure 11

Question 11
Figure 11 is the anteroposterior radiograph of a 20-year-old dancer who fell during his routine and injured
his right foot. What is the most appropriate treatment?

1. Closed reduction and cast


2. Open reduction and internal fixation
3. In situ percutaneous pinning
4. Posterior splint immobilization and controlled ankle motion (CAM) walker ambulation

PREFERRED RESPONSE: 2

DISCUSSION
This patient has sustained a Lisfranc fracture dislocation of the forefoot. To fully restore foot function, an
open reduction and internal fixation should be performed to anatomically reduce this dislocation. Closed
reduction is unlikely to restore normal foot biomechanics and would likely result in delayed arthritis and
joint incongruity. Posterior splint immobilization, CAM walker ambulation, and in situ percutaneous
pinning will not adequately reduce the fracture and restore normal function to the foot.

RECOMMENDED READINGS
Watson TS, Shurnas PS, Denker J. Treatment of Lisfranc joint injury: current concepts. J Am Acad Orthop
Surg. 2010 Dec;18(12):718-28. Review. PubMed PMID: 21119138.
Stavlas P, Roberts CS, Xypnitos FN, Giannoudis PV. The role of reduction and internal fixation of
Lisfranc fracture-dislocations: a systematic review of the literature. Int Orthop. 2010 Dec;34(8):1083-91.
Epub 2010 Aug 5. Review. PubMed PMID: 20683593.

2013 American Academy of Orthopaedic Surgeons 2013 Sports Medicine Self-Assessment Examination
20 American Academy of Orthopaedic Surgeons

Question 12
What factor highly correlates with poor outcomes after surgery for femoroacetabular impingement?

1. Age younger than 20


2. Degenerative arthritis
3. Prominence of the femoral head in cam impingement
4. The patient is a professional athlete

PREFERRED RESPONSE: 2

DISCUSSION
A systematic review of case studies looking at the results of surgical treatment for femoroacetabular
impingement showed good results for most patients, with the exception of those with preoperative
radiographs showing osteoarthritis or Outerbridge grade III or grade IV cartilage damage noted
intraoperatively. Both Byrd and Jones and Philippon and associates have shown good surgical results
for this condition among professional athletes. Likewise, Fabricant and associates demonstrated good
surgical results among adolescent patients with an average age of 17.6 years.

RECOMMENDED READINGS
Ng VY, Arora N, Best TM, Pan X, Ellis TJ. Efficacy of surgery for femoroacetabular impingement: a
systematic review. Am J Sports Med. 2010 Nov;38(11):2337-45. Epub 2010 May 20. Review. PubMed
PMID: 20489213.
Philippon M, Schenker M, Briggs K, Kuppersmith D. Femoroacetabular impingement in 45 professional
athletes: associated pathologies and return to sport following arthroscopic decompression. Knee Surg
Sports Traumatol Arthrosc. 2007 Jul;15(7):908-14. Epub 2007 May 4. PubMed PMID: 17479250.
Byrd JW, Jones KS. Arthroscopic management of femoroacetabular impingement in athletes. Am J Sports
Med. 2011 Jul;39 Suppl:7S-13S. PubMed PMID: 21709026.
Fabricant PD, Heyworth BE, Kelly BT. Hip arthroscopy improves symptoms associated with FAI in
selected adolescent athletes. Clin Orthop Relat Res. 2012 Jan;470(1):261-9. Epub 2011 Aug 11. PubMed
PMID: 21833657.

CLINICAL SITUATION FOR QUESTIONS 13 THROUGH 16


A 40-year-old man who is a manual laborer has had 3 years of worsening medial-sided left knee pain that
has inhibited his ability to work. He reports undergoing a left subtotal medial menisectomy 10 years ago.
He has been treated with nonsteroidal anti-inflammatory drugs and 2 different corticosteroids, with the
most recent injection given 1 month ago. Each injection provided him with a few weeks of pain control.
His medical history is unremarkable and he has smoked 20 cigarettes per day for the last 15 years. His
body mass index (BMI) is 22. On examination, he has varus alignment of the involved leg and medial
joint line tenderness and no lateral or patellofemoral pain. His knee range of motion is 3 degrees shy of
full extension to 130 degrees of flexion. He has negative Lachman and posterior drawer test results. He
demonstrates no lateral thrust with ambulation.

2013 American Academy of Orthopaedic Surgeons 2013 Sports Medicine Self-Assessment Examination
2013 Sports Medicine Self-Assessment Examination Answer Book 21

Question 13
What imaging study is most appropriate to determine treatment options for this patient?

1. Full-length weight-bearing radiographs of both legs


2. MRI scan of the left knee
3. CT scan of the left knee
4. Ultrasound of the left leg

PREFERRED RESPONSE: 1

Question 14
What is the most appropriate next step in treatment?

1. Repeat corticosteroid injection


2. Trial of a medial unloader brace
3. MRI scan of the knee to evaluate for recurrent medial meniscus tear
4. Referral to pain management

PREFERRED RESPONSE: 2

Question 15
The patient is provided with a medial unloader brace that provides substantial pain relief and he is able to
work while wearing the brace. After 4 months he returns to work and says that while the brace enables
him to work, it is uncomfortable. Consequently, his symptoms return when he is not wearing the brace
and he is requesting a surgical intervention for his problem. What is the most appropriate surgical
treatment?

1. Valgus-producing high tibial osteotomy (VPHTO)


2. Repeat knee arthroscopy
3. Total knee arthroplasty (TKA)
4. Medial meniscus transplant

PREFERRED RESPONSE: 1

2013 American Academy of Orthopaedic Surgeons 2013 Sports Medicine Self-Assessment Examination
22 American Academy of Orthopaedic Surgeons

Question 16
The patient is offered a VPHTO. What aspect of his history will determine the most appropriate VPHTO
technique?

1. Prior arthroscopy
2. Current smoking history
3. BMI of 22
4. Age of 40

PREFERRED RESPONSE: 2

DISCUSSION FOR QUESTIONS 13 THROUGH 16


This patient has a classic presentation of postmeniscectomy medial compartment arthritis. The
appropriate diagnostic study is weight-bearing radiographs to confirm the diagnosis. An MRI scan
will reveal medial compartment arthritis but will not provide information about alignment. A CT scan
would be appropriate to detect an occult fracture; however, this condition is not suspected in this clinical
scenario. An ultrasound can provide information about fluid collection around the knee or a deep vein
thrombosis; however, these conditions also are not suspected in this clinical scenario.

Because the patient has a correctable deformity (gaps 3 mm with valgus stress) and his symptoms are
localized to the involved compartment, a trial of a medial unloader brace is appropriate both diagnostically
and therapeutically. If unloading the medial compartment resolves the patients symptoms, he would be
an excellent candidate for an osteotomy. An MRI scan may be obtained to evaluate ligamentous integrity
or to evaluate degenerative involvement of the lateral and patellofemoral compartment for presurgical
planning of an osteotomy; however, the integrity of the medial meniscus has no clinical importance in
a patient with severe medial compartment arthritis. A repeat corticosteroid injection is not indicated
within 1 month of his last injection, and referral to pain management is not appropriate with other options
available to help this patient.

A VPHTO is the appropriate intervention considering the patients young age, high-functional occupation,
examination, radiographic findings, and response to medial unloader bracing. A revision knee arthroscopy
would be appropriate for a recurrent medial meniscus tear, but not in a patient with severe medial
compartment arthritis. The patients young age and high functional requirements are contraindications to
TKA. The presence of severe arthritis is a contraindication to medial meniscus transplant.

The patient is a candidate for a VPHTO. The technical options include a medial opening-wedge or a
lateral closing-wedge osteotomy. Both techniques have advantages and disadvantages; however, a medial
opening-wedge osteotomy is contraindicated in a smoker because of concern for nonunion. As a result,
current smoking history is the only factor listed that would influence the technique used. The history of
prior arthroscopy has no relevance in the decision about which type of osteotomy is appropriate. Normal
BMI is between 18.5 and 24.9, so this patients BMI is considered normal and would not affect the
surgical technique (if this patient were obese, a lateral closing-wedge osteotomy would be considered, but
this is controversial). His age of 40 is an indication for HTO but does not influence technique.

2013 American Academy of Orthopaedic Surgeons 2013 Sports Medicine Self-Assessment Examination
2013 Sports Medicine Self-Assessment Examination Answer Book 23

RECOMMENDED READINGS FOR QUESTIONS 13 THROUGH 16


Rossi R, Bonasia DE, Amendola A. The role of high tibial osteotomy in the varus knee. J Am Acad Orthop
Surg. 2011 Oct;19(10):590-9. Review. PubMed PMID: 21980024.
Sprenger TR, Doerzbacher JF. Tibial osteotomy for the treatment of varus gonarthrosis. Survival and
failure analysis to twenty-two years. J Bone Joint Surg Am. 2003 Mar;85-A(3):469-74. Erratum in: J Bone
Joint Surg Am. 2003 May 85-A(5):912. PubMed PMID: 12637433.
Amendola A. Knee osteotomy and meniscal transplantation: indications, technical considerations, and
results. Sports Med Arthrosc. 2007 Mar;15(1):32-8. Review. PubMed PMID: 17301700.
W-Dahl A, Toksvig-Larsen S. Cigarette smoking delays bone healing: a prospective study of 200 patients
operated on by the hemicallotasis technique. Acta Orthop Scand. 2004 Jun;75(3):347-51. PubMed PMID:
15260429.

END OF SERIES

Question 17
When reconstructing the anterior cruciate ligament (ACL), what is the most common source of potential
autograft failure?

1. Graft choice
2. Tunnel position
3. Tibial fixation
4. Femoral fixation

PREFERRED RESPONSE: 2

DISCUSSION
Technical failure is the most common reason for ACL reconstruction failure. Tunnel position is the most
frequent cause for technical failure. Malpositioning of the tunnel affects the length of the graft, causing
either decreased range of motion or increased graft laxity. Although graft choice is an important factor
when planning an ACL reconstruction, overall outcomes with autograft tissues are fairly similar. Fixation
of the graft at the femoral or tibial end is not as important as tunnel position.

RECOMMENDED READINGS
Battaglia TC, Miller MD. Management of bony deficiency in revision anterior cruciate ligament
reconstruction using allograft bone dowels: surgical technique. Arthroscopy. 2005 Jun;21(6):767. PubMed
PMID: 15944645.
Beynnon BD, Johnson RJ, Fleming BC, Kannus P, Kaplan M, Samani J, Renstrm P. Anterior cruciate
ligament replacement: comparison of bone-patellar tendon-bone grafts with two-strand hamstring grafts.
A prospective, randomized study. J Bone Joint Surg Am. 2002 Sep;84-A(9):1503-13. PubMed PMID:
12208905.

2013 American Academy of Orthopaedic Surgeons 2013 Sports Medicine Self-Assessment Examination
24 American Academy of Orthopaedic Surgeons

CLINICAL SITUATION FOR QUESTIONS 18 THROUGH 20


A 25-year-old healthy woman injured her left knee while playing professional soccer. She has never
injured this knee before. Examination 2 days after the injury occurred reveals the following: a moderate
effusion, a positive Lachman test result, and mild lateral tenderness. Range of motion is between 20
degrees and 70 degrees. Radiographs reveal no fracture. An MRI scan reveals a complete rupture of the
anterior cruciate ligament (ACL), an effusion, and bone bruises of the lateral femoral condyle and lateral
tibial plateau. No meniscal tear is seen. The patient would like to continue playing at the professional
level.

Question 18
What is the next treatment step?

1. Immobilization of the knee for 6 weeks, followed by rehabilitation and delayed ACL
reconstruction
2. Immediate ACL reconstruction
3. Immediate rehabilitation for 6 months followed by ACL reconstruction if the patient is
unstable in a brace
4. Immediate rehabilitation with delayed ACL reconstruction (when the athlete obtains full knee
range of motion)

PREFERRED RESPONSE: 4

Question 19
What is this patients risk for developing osteoarthritis (OA) of the knee?

1. There is no risk for development of knee OA after reconstruction of the ligament.


2. There is no risk for development of knee OA after a double-bundle ACL reconstruction.
3. There is no evidence that ACL reconstruction reduces the incidence of knee OA.
4. There is 100% likelihood that she will develop knee OA after single-bundle ACL
reconstruction.

PREFERRED RESPONSE: 3

2013 American Academy of Orthopaedic Surgeons 2013 Sports Medicine Self-Assessment Examination
2013 Sports Medicine Self-Assessment Examination Answer Book 25

Question 20
The patient asks if something about her anatomy has resulted in this injury. ACL anatomy differs between
men and women in what manner?

1. There is no significant difference in ACL anatomy between men and women.


2. A womans ACL has a smaller cross-sectional area.
3. The cross-sectional area of a womans ACL is larger.
4. The intercondylar notch is wider in women than in men.

PREFERRED RESPONSE: 2

DISCUSSION FOR QUESTIONS 18 THROUGH 20


This patient has the clinical findings of an ACL rupture that is confirmed on MRI scan. She is a
professional athlete and would like to return to her sport. Immediate ACL reconstruction in the setting
of a knee with limited motion carries an increased risk for postsurgical stiffness. Delayed surgery after
the patient regains range of motion is the preferred response. It has been shown that a womans ACL is
smaller in the cross-sectional area.

RECOMMENDED READINGS FOR QUESTIONS 18 THROUGH 20


Phelan DT, Cohen AB, Fithian DC. Complications of anterior cruciate ligament reconstruction. Instr
Course Lect. 2006;55:465-74. Review. PubMed PMID: 16958481.
Scanlan SF, Chaudhari AM, Dyrby CO, Andriacchi TP. Differences in tibial rotation during walking in
ACL reconstructed and healthy contralateral knees. J Biomech. 2010 Jun 18;43(9):1817-22. Epub 2010
Feb 23. PubMed PMID: 20181339
Crawford C, Nyland J, Landes S, Jackson R, Chang HC, Nawab A, Caborn DN. Anatomic double bundle
ACL reconstruction: a literature review. Knee Surg Sports Traumatol Arthrosc. 2007 Aug;15(8):946-64;
discussion 945. Epub 2007 May 30. Review. PubMed PMID: 17534599.
Lipps DB, Oh YK, Ashton-Miller JA, Wojtys EM. Morphologic characteristics help explain the gender
difference in peak anterior cruciate ligament strain during a simulated pivot landing. Am J Sports Med.
2012 Jan;40(1):32-40. Epub 2011 Sep 14. PubMed PMID: 21917612.
Alentorn-Geli E, Myer GD, Silvers HJ, Samitier G, Romero D, Lzaro-Haro C, Cugat R. Prevention
of non-contact anterior cruciate ligament injuries in soccer players. Part 1: Mechanisms of injury and
underlying risk factors. Knee Surg Sports Traumatol Arthrosc. 2009 Jul;17(7):705-29. Epub 2009 May 19.
Review. PubMed PMID: 19452139.
Buoncristiani AM, Tjoumakaris FP, Starman JS, Ferretti M, Fu FH. Anatomic double-bundle anterior
cruciate ligament reconstruction. Arthroscopy. 2006 Sep;22(9):1000-6. PubMed PMID: 16952731.

END OF SERIES

2013 American Academy of Orthopaedic Surgeons 2013 Sports Medicine Self-Assessment Examination
26 American Academy of Orthopaedic Surgeons

Figure 21

Question 21
Figure 21 is the radiograph of a 31-year-old man who had left shoulder pain after a fall during a
snowboarding jump. Residual displacement of 5 mm after closed reduction is most likely to result in
which of the following?

1. Nonunion
2. Osteonecrosis
3. Altered rotator cuff mechanics
4. Normal shoulder function

PREFERRED RESPONSE: 3

DISCUSSION
Humerus fractures account for 11% of all fractures among snowboarders and are the second-most-
common upper-extremity fracture after radius fractures (48%). Surgical fixation is recommended for
fractures with residual displacement greater than 5 mm, or 3 mm in active patients involved in frequent
overhead activity. Malunion can result in a mechanical block to shoulder abduction or external rotation
and altered rotator cuff mechanics, causing weakness. A rich arterial network provides a favorable healing
environment for greater tuberosity fractures. Consequently, nonunion and osteonecrosis are uncommon.

RECOMMENDED READINGS
Bissell BT, Johnson RJ, Shafritz AB, Chase DC, Ettlinger CF. Epidemiology and risk factors of humerus
fractures among skiers and snowboarders. Am J Sports Med. 2008 Oct;36(10):1880-8. Epub 2008 Jul 1.
PubMed PMID: 18593842.
George MS. Fractures of the greater tuberosity of the humerus. J Am Acad Orthop Surg. 2007
Oct;15(10):607-13. Review. PubMed PMID: 17916784.

2013 American Academy of Orthopaedic Surgeons 2013 Sports Medicine Self-Assessment Examination
2013 Sports Medicine Self-Assessment Examination Answer Book 27

Question 22
What strategy has proven most effective in preventing transmission of methicillin-resistant Staphylococcus
aureus among teammates?

1. Separate players with infections in a separate locker room or changing area.


2. Treat teammates of the infected player with prophylactic antibiotics.
3. Cover any skin lesions with occlusive dressing during sporting activity.
4. Ban players with infections from any team event.

PREFERRED RESPONSE: 3

DISCUSSION
Prevention is the key to controlling infections among athletes. Proper hygiene is critical and should
mandate showering, hand washing, wearing breathable clothing, and shower sandals. The sharing of
towels or athletic equipment should be forbidden. Daily skin surveillance by athletes, trainers, and
physicians can allow early recognition and treatment initiation during the early stages of infection,
limiting risk for further transmission. Additionally, disinfecting shared equipment, covering lesions with
occlusive dressing during sporting activity, and restricting the contact activities of infected athletes can
limit risk for an infectious outbreak among teammates.

RECOMMENDED READINGS
Kirkland EB, Adams BB. Methicillin-resistant Staphylococcus aureus and athletes. J Am Acad Dermatol.
2008 Sep;59(3):494-502. Epub 2008 Jun 11. Review. PubMed PMID: 18550208.
Sedgwick PE, Dexter WW, Smith CT. Bacterial dermatoses in sports. Clin Sports Med. 2007
Jul;26(3):383-96. Review. PubMed PMID: 17826190.
Rihn JA, Michaels MG, Harner CD. Community-acquired methicillin-resistant staphylococcus aureus:
an emerging problem in the athletic population. Am J Sports Med. 2005 Dec;33(12):1924-9. Review.
PubMed PMID: 16314668.

2013 American Academy of Orthopaedic Surgeons 2013 Sports Medicine Self-Assessment Examination
28 American Academy of Orthopaedic Surgeons

Figure 23

Question 23
Figure 23 is the T2 axial MRI scan of a 21-year-old man who was injured while playing for his college
football team. His pain was aggravated with blocking maneuvers and alleviated with rest, and he had to
stop playing because of the pain. What examination maneuver most likely will reproduce his pain?

1. Forward elevation in the scapular plane


2. External rotation and abduction
3. Flexion, adduction, and internal rotation
4. Flexion and abduction

PREFERRED RESPONSE: 3

DISCUSSION
This patient has a mechanism of injury and MRI consistent with a posterior labral tear and posterior
instability. Flexion, adduction, and internal rotation produce a net posterior vector on the glenohumeral
joint and should reproduce this patient's symptoms. Pain or instability with the arm elevated in the
scapular plane describes an impingement sign. Pain or instability with the arm in external rotation and
abduction describes the apprehension sign. Pain or instability with the arm in flexion and abduction is a
nonspecific finding.

RECOMMENDED READINGS
Bradley JP, Forsythe B, Mascarenhas R. Arthroscopic management of posterior shoulder instability:
diagnosis, indications, and technique. Clin Sports Med. 2008 Oct;27(4):649-70. Review. PubMed PMID:
19064149.
Tjoumakaris FP, Bradley JP. Posterior shoulder instability. In: Galatz LM, ed. Orthopaedic Knowledge
Update: Shoulder and Elbow 3. Rosemont, IL: American Academy of Orthopaedic Surgeons; 2008:313-
320.

2013 American Academy of Orthopaedic Surgeons 2013 Sports Medicine Self-Assessment Examination
2013 Sports Medicine Self-Assessment Examination Answer Book 29

CLINICAL SITUATION FOR QUESTIONS 24 AND 25


During the third quarter of a high school football game, a 16-year-old running back gets tackled and
limps off the field. During the initial sideline evaluation, he has tenderness on the right iliac crest. He is
a little dizzy, has a headache, and tells you, I need to get back in the game to help the team score before
halftime.

Question 24
How can this scenario be managed most effectively?

1. Initiate rest, ice the iliac crest, and return to play when he is not limping.
2. Initiate rest, ice the iliac crest, and return to play after 20 minutes.
3. Keep the player on the sideline, perform a cognitive evaluation, and repeat the physical
assessment.
4. Keep the player out of the game and send him emergently to the hospital for imaging.

PREFERRED RESPONSE: 3

Question 25
Sideline examination of this patient showed no cervical pain or tenderness; motor and sensory function
were normal; and his pupils were equal, round, and reactive. He was alert and oriented to the score of
game, time on the clock, and current quarter of play. His iliac crest had mild tenderness but no swelling
or crepitus. The player states that he has a slight headache and is no longer dizzy. What is the most
appropriate treatment?

1. Return him to the game and observe his play closely.


2. Do not return to the game and do not allow play for the remainder of the season.
3. Do not return to the game and begin a graduated return-to-play protocol for future games.
4. Perform a sideline noncontact exercise testing examination and return him to the game if he is
asymptomatic.

PREFERRED RESPONSE: 3

2013 American Academy of Orthopaedic Surgeons 2013 Sports Medicine Self-Assessment Examination
30 American Academy of Orthopaedic Surgeons

DISCUSSION FOR QUESTIONS 24 AND 25


Although this player limps off the field, the fact that he felt dizzy, had a headache, and did not initially
recognize that he was playing in the third quarter indicates that he sustained a concussion. The player
should be kept out of the game until a cognitive examination and repeat physical assessment is completed.
Even if his physical symptoms have resolved, a certain period of time has expired, or he states that he is
ready, he should not be returned to play prior to this assessment. Sending the patient to an emergency
department should be considered only after this assessment and appropriate initial sideline treatment is
initiated. The Consensus Statement on Concussion in Sport recommends that no athlete with concussion
symptoms be returned to same-day play. This patient still has a slight headache, but even if this resolved
he should not return to the game. Adolescents and high school athletes may have neurophysiological
deficits that may not be evident on the sideline, or they may have a delayed onset of symptoms. A
graduated return to play for future games is recommended.

RECOMMENDED READINGS FOR QUESTIONS 24 AND 25


Putukian M. The acute symptoms of sport-related concussion: diagnosis and on-field management. Clin
Sports Med. 2011 Jan;30(1):49-61, viii. Review. PubMed PMID: 21074081.
McCrory P, Meeuwisse W, Johnston K, Dvorak J, Aubry M, Molloy M, Cantu R. Consensus statement on
concussion in sport - the Third International Conference on Concussion in Sport held in Zurich, November
2008. Phys Sportsmed. 2009 Jun;37(2):141-59. Review. PubMed PMID: 20048521.
d'Hemecourt P. Subacute symptoms of sports-related concussion: outpatient management and return to
play. Clin Sports Med. 2011 Jan;30(1):63-72, viii. Review. PubMed PMID: 21074082.

END OF SERIES

CLINICAL SITUATION FOR QUESTIONS 26 THROUGH 29


A 32-year-old woman has a 2-year history of progressively worsening right groin pain that is exacerbated
by activity. She reports no traumatic injury and an extensive work-up by her gynecologist has ruled out
an intrapelvic source of her pain. The patient is a recreational athlete and exercises regularly in the gym.
The pain is preventing her from performing these activities. She reports no catching or locking symptoms.
Her examination reveals a physically fit female (BMI of 20) with limited right hip range of motion. She
has no tenderness to palpation around the hip. While lying supine and bringing her hip into progressive
flexion with internal rotation and adduction, her groin pain is reproduced. She has normal limb lengths
and demonstrates weakness secondary to pain with hip flexion on the affected side.

Question 26
What is the most likely cause of this patients groin pain?

1. Femoroacetabular impingement (FAI)


2. Osteoarthritis of the sacroiliac joint
3. Intra-articular loose body
4. Trochanteric bursitis

PREFERRED RESPONSE: 1
2013 American Academy of Orthopaedic Surgeons 2013 Sports Medicine Self-Assessment Examination
2013 Sports Medicine Self-Assessment Examination Answer Book 31

Question 27
The patient is enrolled in physical therapy for 6 weeks with little improvement of her hip symptoms.
What is the next most appropriate diagnostic test to determine the presence of an associated acetabular
labral tear in this patient?

1. Diagnostic arthroscopy of the hip


2. MRI scan of the hip
3. MRI arthrogram of the hip
4. Ultrasound of the hip

PREFERRED RESPONSE: 3

Question 28
The study obtained in question 27 confirms the presence of an anterosuperior acetabular labral tear and
pincer morphology of the acetabulum. What is the most likely location of a chondral injury associated
with these findings?

1. Posteroinferior acetabulum
2. Posterosuperior acetabulum
3. Femoral head above the fovea
4. Femoral head below the fovea

PREFERRED RESPONSE: 1

Question 29
The patient experienced little improvement with activity modification and physical therapy. An intra-
articular corticosteroid injection provides excellent but short-lived pain control. She requests surgical
treatment for her hip and she is counseled regarding arthroscopy and consent is obtained. Intraoperatively,
a capsulolabral separation is observed with an underlying pincer lesion. No articular cartilage injury is
seen. What treatment is most appropriate considering these findings?

1. Suture anchor repair of the labral tear and no bony resection


2. Suture anchor repair of the labral tear and bony resection of the pincer lesion
3. Debridement of the labral tear and bony resection of the pincer lesion
4. Debridement of the labral tear with no bony resection of the pincer lesion

PREFERRED RESPONSE: 2

2013 American Academy of Orthopaedic Surgeons 2013 Sports Medicine Self-Assessment Examination
32 American Academy of Orthopaedic Surgeons

DISCUSSION FOR QUESTIONS 26 THROUGH 29


The clinical scenario, examination, and MRI scans are consistent with a pincer-type FAI. The decreased
range of motion is secondary to the pain produced by the continued abutment of the femoral head against
the anterosuperior acetabulum. Flexing the hip while internally rotating and adducting the leg recreates
this contact and is typically painful. No clinical signs suggest sacroiliac joint arthritis, an intra-articular
loose body, or trochanteric bursitis, although these are all diagnoses that should be considered in a patient
with a painful hip. The most sensitive and specific study to detect an acetabular labral tear is an MRI
arthrogram of the hip. This study should be obtained in this patient to evaluate the labrum as well as
the status of the articular cartilage. An MRI scan without intra-articular contrast is not as sensitive as
an arthrogram. An ultrasound can provide a dynamic assessment of the hip and help in the setting of a
snapping hip; however, this study is not reliable to determine the presence of a labral tear. In the setting
of pincer FAI, the forced leverage of the anterosuperior femoral head upon the anterior acetabulum results
in abnormal forces against the posteroinferior acetabulum. This continued force can lead to a chondral
lesion in this location know as a counter-coup injury. Chondral lesions of the femoral head are rare in
the setting of pincer FAI. The posterosuperior quadrant does not experience increased force and rarely
sustains chondral injuries. The patient is a young, active individual with no pre-existing degenerative
changes, so repair of the tear with bony resection of the pincer lesion is the most appropriate treatment.
A capsulolabral detachment should be repaired because these tears can heal and the labrum functions as
a seal, preventing egress of synovial fluid from the joint space. If the pincer lesion is not resected, the
patient will continue to experience abnormal contact and the repair will likely fail. There is no evidence
that the patient has a cam impingement, and recontouring of the femoral head/neck junction is not
appropriate. Simple debridement should be reserved for intrasubstance tears of the labrum, which would
not be expected to heal with repair.

RECOMMENDED READINGS FOR QUESTIONS 26 THROUGH 29


Ganz R, Parvizi J, Beck M, Leunig M, Ntzli H, Siebenrock KA. Femoroacetabular impingement: a cause
for osteoarthritis of the hip. Clin Orthop Relat Res. 2003 Dec;(417):112-20. Review. PubMed PMID:
14646708.
Leunig M, Podeszwa D, Beck M, Werlen S, Ganz R. Magnetic resonance arthrography of labral disorders
in hips with dysplasia and impingement. Clin Orthop Relat Res. 2004 Jan;(418):74-80. PubMed PMID:
15043096.
Leunig M, Werlen S, Ungersbck A, Ito K, Ganz R. Evaluation of the acetabular labrum by MR
arthrography. J Bone Joint Surg Br. 1997 Mar;79(2):230-4. Erratum in: J Bone Joint Surg Br 1997
Jul;79(4):693. PubMed PMID: 9119848.
Parvizi J, Leunig M, Ganz R. Femoroacetabular impingement. J Am Acad Orthop Surg. 2007
Sep;15(9):561-70. PubMed PMID: 17761612.
Konan S, Rayan F, Meermans G, Witt J, Haddad FS. Validation of the classification system for acetabular
chondral lesions identified at arthroscopy in patients with femoroacetabular impingement. J Bone Joint
Surg Br. 2011 Mar;93(3):332-6. Review. PubMed PMID: 21357954.
Schoenecker PL, Clohisy JC, Millis MB, Wenger DR. Surgical management of the problematic hip in
adolescent and young adult patients. J Am Acad Orthop Surg. 2011 May;19(5):275-86. Review. PubMed
PMID: 21536627.
Matsuda DK, Carlisle JC, Arthurs SC, Wierks CH, Philippon MJ. Comparative systematic review of the
open dislocation, mini-open, and arthroscopic surgeries for femoroacetabular impingement. Arthroscopy.
2011 Feb;27(2):252-69. Review. PubMed PMID: 21266276.

2013 American Academy of Orthopaedic Surgeons 2013 Sports Medicine Self-Assessment Examination
2013 Sports Medicine Self-Assessment Examination Answer Book 33

Petersen W, Petersen F, Tillmann B. Structure and vascularization of the acetabular labrum with regard to
the pathogenesis and healing of labral lesions. Arch Orthop Trauma Surg. 2003 Jul;123(6):283-8. Epub
2003 Jun 7. PubMed PMID: 12802599.
Safran MR. The acetabular labrum: anatomic and functional characteristics and rationale for surgical
intervention. J Am Acad Orthop Surg. 2010 Jun;18(6):338-45. Review. PubMed PMID: 20511439.

END OF SERIES

Figure 30a Figure 30b Figure 30c

Figure 30d Figure 30e

CLINICAL SITUATION FOR QUESTIONS 30 THROUGH 32


Figures 30a and 30b are the radiographs of a 20-year-old college multisport athlete who has had
longstanding pain in his left hip. He denies any specific event that initiated his pain, but he notes that he
had hip problems when he was an infant. He denies pain with activities of daily living, but he believes his
pain is increasingly limiting his ability to exercise. He localizes the pain to his groin. He denies low-back
or buttock pain or pain that radiates down his leg.

2013 American Academy of Orthopaedic Surgeons 2013 Sports Medicine Self-Assessment Examination
34 American Academy of Orthopaedic Surgeons

Question 30
What examination findings are most consistent with the pathology seen in the radiographs?

1. Pain with resisted hip flexion


2. Pain with a half sit-up, plus tenderness at the pubic ramus
3. Pain with a combination of hip flexion, adduction, and internal rotation
4. Tenderness to palpation at the greater trochanter

PREFERRED RESPONSE: 3

Question 31
What is the most likely diagnosis for the source of this patients pain?

1. Cam-type femoroacetabular impingement


2. Pincer-type femoroacetabular impingement
3. Hip flexor strain
4. Athletic pubalgia

PREFERRED RESPONSE: 1

Question 32
Images from an MRI scan of this patients left hip are shown in Figure 30c through 30e. What is the most
likely cause of his acute pain?

1. Significant cartilage loss on the acetabulum


2. Labral tear
3. Femoral neck stress fracture
4. Tendonopathy of the rectus femoris

PREFERRED RESPONSE: 2

2013 American Academy of Orthopaedic Surgeons 2013 Sports Medicine Self-Assessment Examination
2013 Sports Medicine Self-Assessment Examination Answer Book 35

DISCUSSION FOR QUESTIONS 30 THROUGH 32


This patient has cam-type femoroacetabular impingement. Decreased internal rotation and a positive
impingement test (forced flexion, adduction, and internal rotation) are classic findings. The lack of
pain with resisted hip flexion makes hip flexor strain unlikely, and the lack of tenderness at the greater
trochanter renders trochanteric bursitis unlikely. Although athletic pubalgia can be a source of long-
standing groin pain, he lacks the pain with a resisted sit-up and tenderness along the pubic ramus that
is frequently noted in patients with pubalgia. His radiographs reveal a focal femoral neck prominence
consistent with cam impingement, although pistol grip deformities and flattening of the lateral femoral
head are often present as well. His MRI scan shows a labral tear, which is common in cam impingement.
Surgical treatment for cam impingement can be effective for symptomatic patients. Even among high-
level athletes, open surgical dislocation of the hip has been shown to have good results. Most patients
with cam impingement can be treated with arthroscopic osteoplasty and achieve results comparable
to those realized with open surgical dislocation. The literature describes success in terms of athletes
returning to sports (even professional athletes) to be approximately 90% after arthroscopic treatment.
Byrd and Jones described 5 patients who developed transient neurapraxias that resolved uneventfully.
The patients in his series who had concomitant microfracture had a 92% return to sports within the
follow-up period. Cam impingement has long been thought to be associated with a history of a slipped
capital femoral epiphysis. The capitis in these patients is displaced posteriorly, resulting in a prominent
anterior femoral neck and decreased hip internal rotation. Pincer impingement is associated with a
deep acetabulum, such as protrusion acetabula and acetabular retroversion. A patient who underwent a
periacetabular osteotomy can develop a more retroverted acetabulum as well.

RECOMMENDED READINGS FOR QUESTIONS 30 THROUGH 32


Schoenecker PL, Clohisy JC, Millis MB, Wenger DR. Surgical management of the problematic hip in
adolescent and young adult patients. J Am Acad Orthop Surg. 2011 May;19(5):275-86. Review. PubMed
PMID: 21536627.
Minnich JM, Hanks JB, Muschaweck U, Brunt LM, Diduch DR. Sports hernia: diagnosis and treatment
highlighting a minimal repair surgical technique. Am J Sports Med. 2011 Jun;39(6):1341-9. Epub 2011
Apr 19. Review. PubMed PMID: 21505079.
Nepple JJ, Carlisle JC, Nunley RM, Clohisy JC. Clinical and radiographic predictors of intra-articular hip
disease in arthroscopy. Am J Sports Med. 2011 Feb;39(2):296-303. Epub 2010 Nov 23. PubMed PMID:
21098820.
Naal FD, Miozzari HH, Wyss TF, Ntzli HP. Surgical hip dislocation for the treatment of
femoroacetabular impingement in high-level athletes. Am J Sports Med. 2011 Mar;39(3):544-50. Epub
2010 Dec 20. PubMed PMID: 21173196.
Bedi A, Zaltz I, De La Torre K, Kelly BT. Radiographic comparison of surgical hip dislocation and hip
arthroscopy for treatment of cam deformity in femoroacetabular impingement. Am J Sports Med. 2011
Jul;39 Suppl:20S-8S. PubMed PMID: 21709028.
Philippon M, Schenker M, Briggs K, Kuppersmith D. Femoroacetabular impingement in 45 professional
athletes: associated pathologies and return to sport following arthroscopic decompression. Knee Surg
Sports Traumatol Arthrosc. 2007 Jul;15(7):908-14. Epub 2007 May 4. PubMed PMID: 17479250.
Byrd JW, Jones KS. Arthroscopic management of femoroacetabular impingement in athletes. Am J Sports
Med. 2011 Jul;39 Suppl:7S-13S. PubMed PMID: 21709026.
Byrd JW. Femoroacetabular impingement in athletes, part 1: cause and assessment. Sports Health. 2010
Jul;2(4):321-33. PubMed PMID: 23015955.

END OF SERIES
2013 American Academy of Orthopaedic Surgeons 2013 Sports Medicine Self-Assessment Examination
36 American Academy of Orthopaedic Surgeons

Question 33
A 25-year-old recreational soccer player has recurrent shoulder dislocations. He first dislocated his
shoulder playing football in high school, was treated in a sling for 6 weeks, and returned to play for the
remainder of the season. He did well until 2 years later when he reinjured the shoulder. He says that his
shoulder dislocates with little injury and always feels loose. Examination reveals anterior instability
and an MR arthrogram reveals an anterior-inferior labral tear and surgical treatment is recommended. He
inquires about the benefits of arthroscopic vs open procedure. Which of the following statements reflects
an advantage associated with arthroscopic procedures compared to open stabilization?

1. Range of motion might be slightly better after an arthroscopic procedure.


2. Rate of recurrent instability is lower after an arthroscopic procedure.
3. Rates of return to work are higher after an arthroscopic procedure.
4. Rates of return to sports are higher after an arthroscopic procedure.

PREFERRED RESPONSE: 1

DISCUSSION
There is much debate in the literature regarding optimal techniques for treatment of shoulder instability.
Most studies have suggested a slightly better range of motion of the shoulder after an arthroscopic repair.
Recurrent instability rates have been slightly higher with arthroscopic procedures in some studies, while
others show the rates are not statistically different. Return to work and/or sports has been shown to be
equal or slightly better with open procedures.

RECOMMENDED READINGS
Fabbriciani C, Milano G, Demontis A, Fadda S, Ziranu F, Mulas PD. Arthroscopic versus open treatment
of Bankart lesion of the shoulder: a prospective randomized study. Arthroscopy. 2004 May;20(5):456-62.
PubMed PMID: 15122134.
Lenters TR, Franta AK, Wolf FM, Leopold SS, Matsen FA 3rd. Arthroscopic compared with open repairs
for recurrent anterior shoulder instability. A systematic review and meta-analysis of the literature. J Bone
Joint Surg Am. 2007 Feb;89(2):244-54. Review. PubMed PMID: 17272436.
Cole BJ, L'Insalata J, Irrgang J, Warner JJ. Comparison of arthroscopic and open anterior shoulder
stabilization. A two to six-year follow-up study. J Bone Joint Surg Am. 2000 Aug;82-A(8):1108-14.
PubMed PMID: 10954100.

2013 American Academy of Orthopaedic Surgeons 2013 Sports Medicine Self-Assessment Examination
2013 Sports Medicine Self-Assessment Examination Answer Book 37

Figure 34a Figure 34b

Question 34
Figures 34a and 34b are the radiographs of a 38-year-old woman who had increasing left hip pain with
activity. She noted no lower back or buttock pain and no pain along her lateral thigh. The pain usually
only bothers her with running and cycling. Nonsteroidal anti-inflammatory drugs helped initially but are
not relieving her pain now. Examination with the patient supine reveals pain with internal and external
rotation of her hip with her hip and knee in an extended position. With her hip flexed to 90 degrees, she
has internal rotation only to neutral, but full external rotation. What is the most likely diagnosis?

1. Cam-type femoroacetabular impingement


2. Pincer-type femoroacetabular impingement
3. Intra-articular loose body
4. Snapping psoas tendon

PREFERRED RESPONSE: 2

DISCUSSION
This patient has pincer femoroacetabular impingement. Her examination demonstrates pain with
internal and external rotation of the femoral head in the acetabulum, suggesting intra-articular pathology.
She also has a noticeable loss of internal rotation. Her examination findings make the other possible
diagnoses unlikely. Her radiographs show a crossover sign, which suggests overcoverage by the anterior
acetabulum, often the result of acetabular retroversion.

RECOMMENDED READINGS
Byrd JW. Femoroacetabular impingement in athletes, part 1: cause and assessment. Sports Health. 2010
Jul;2(4):321-33. PubMed PMID: 23015955.
Byrd JW, Jones KS. Arthroscopic management of femoroacetabular impingement in athletes. Am J Sports
Med. 2011 Jul;39 Suppl:7S-13S. PubMed PMID: 21709026.

2013 American Academy of Orthopaedic Surgeons 2013 Sports Medicine Self-Assessment Examination
38 American Academy of Orthopaedic Surgeons

Question 35
A 24-year-old former high school wrestler had anterior cruciate ligament (ACL) reconstruction with
hamstring autograft 6 years ago. He now experiences daily instability of his knee with routine activities
including walking. Examination reveals a grade 3+ Lachman with a soft endpoint, varus laxity at 30
degrees, and a positive dial test at 30 degrees that dissipates at 90 degrees of knee flexion. He has mild
medial joint line tenderness. When walking, there is a slight varus thrust. What treatment is most likely to
lead to a successful outcome?

1. Hamstring autograft
2. Revision ACL reconstruction and posterior cruciate ligament (PCL) reconstruction
3. Revision ACL reconstruction and posteromedial corner reconstruction
4. Revision ACL reconstruction and posterolateral corner reconstruction

PREFERRED RESPONSE: 4

DISCUSSION
This patient underwent an ACL reconstruction that has now failed. Based on his examination, he also
has a posterolateral corner injury. Because this concomitant injury was not treated, the patient had undue
strain on his graft, resulting in ultimate failure. Hamstring grafts are as effective as other graft types for
ACL reconstruction. The medial meniscus provides secondary stabilization to the knee; however, this
patient has a missed lateral ligamentous injury, and meniscus tears do not result in the development of
a varus thrust. An unrecognized PCL tear likely results in mild-to-moderate medial and patellofemoral
osteoarthritis without significant lateral laxity and thrust.

RECOMMENDED READINGS
Ricchetti ET, Sennett BJ, Huffman GR. Acute and chronic management of posterolateral corner injuries
of the knee. Orthopedics. 2008 May;31(5):479-88; quiz 489-90. Review. Erratum in: Orthopedics. 2008
Jul;31(7):725. PubMed PMID: 18522011.
Fanelli GC. Treatment of combined anterior cruciate ligament-posterior cruciate ligament-lateral side
injuries of the knee. Clin Sports Med. 2000 Jul;19(3):493-502. Review. PubMed PMID: 10918962.

Question 36
A 49-year-old man is seeking a second opinion for continued knee pain and swelling. He went to his
primary doctor for swelling on top of his knee, and he says his doctor drained some clear fluid. He
noted that his condition improved for about 1 week before the swelling returned. He now has increasing
pain and redness around his kneecap. Examination reveals significant swelling of his prepatellar bursa,
with erythema over the bursa that extends to the surrounding skin. His temperature in the office is
101.7F. What is the next step in treatment for this patient?

1. Initiate oral antibiotics for 7 days.


2. Reaspirate the bursa and inject a corticosteroid.
3. Recommend padding the patella for kneeling and ice.
4. Perform an open bursectomy and start intravenous antibiotics.

PREFERRED RESPONSE: 4
2013 American Academy of Orthopaedic Surgeons 2013 Sports Medicine Self-Assessment Examination
2013 Sports Medicine Self-Assessment Examination Answer Book 39

DISCUSSION
This patient has septic prepatellar bursitis. Padding, cold therapy, compression, nonsteroidal anti-
inflammatory drugs, and aspiration are thought to be acceptable treatments for aseptic prepatellar bursitis;
they have little role in septic bursitis. Antibiotics along with aspiration or placement of a percutaneous
drain have been associated with success in some cases of septic prepatellar bursitis, but the standard
treatment is complete bursectomy with systemic antibiotics.

RECOMMENDED READINGS
Aaron DL, Patel A, Kayiaros S, Calfee R. Four common types of bursitis: diagnosis and management. J
Am Acad Orthop Surg. 2011 Jun;19(6):359-67. Review. PubMed PMID: 21628647.
Knight JM, Thomas JC, Maurer RC. Treatment of septic olecranon and prepatellar bursitis with
percutaneous placement of a suction-irrigation system. A report of 12 cases. Clin Orthop Relat Res. 1986
May;(206):90-3. PubMed PMID: 3011340.

Figure 37a Figure 37b

CLINICAL SITUATION FOR QUESTIONS 37 AND 38


An 18-year-old right-hand-dominant college freshman who is a third baseman has pain in his right
shoulder after using his outstretched right arm for support while diving to catch a low line drive. He
describes pivoting on his right hand and arm while reaching out to make the catch with his left-hand glove.
He had pain in his shoulder but was able to finish the game with some pain while throwing. Five days
later, he experiences popping pain deep in his shoulder that has improved since the injury but continued to
be bothersome deep in the shoulder with higher-velocity throwing. Examination demonstrated decreased
internal rotation, posterior pain with cross-body adduction and posteriorly directed force, and full rotator
cuff strength. Radiograph findings were normal. After 6 weeks of physical therapy (PT), his range
of motion has improved but he continues to experience deep pain with therapy. Examination shows
symmetric range of motion and posterior pain with the jerk test and Kim test. His rotator cuff is strong.

2013 American Academy of Orthopaedic Surgeons 2013 Sports Medicine Self-Assessment Examination
40 American Academy of Orthopaedic Surgeons

Question 37
What is the next step in treatment?

1. Continued PT
2. Subacromial injection
3. CT scan
4. MRI arthrogram

PREFERRED RESPONSE: 4

Question 38
T1-weighted, fat-saturated MRI scans are shown in Figures 37a and 37b. What is the next step in
treatment?

1. Open reduction internal fixation (ORIF)


2. Rotator cuff repair
3. Labrum repair
4. Chondroplasty

PREFERRED RESPONSE: 3

DISCUSSION FOR QUESTIONS 37 AND 38


Examination findings of posterior glenohumeral tenderness, decreased internal rotation, and reproduction
of symptoms with a posterior stress test indicate a posterior shoulder injury or instability. The jerk test,
with the patient seated, positions the arm in forward flexion and internal rotation with elbow flexion. One
hand of the examiner is placed on the patients distal clavicle and scapular spine and the other hand grasps
the elbow. The arm is jerked posteriorly while the shoulder girdle is jerked anteriorly, which creates pain
as the posteriorly subluxated humeral head relocates into the glenoid fossa. During the Kim test, the
patient is seated with the arm in 90 degrees of abduction. While the arm is elevated 45 degrees diagonally
(forward flexion and adduction), the examiner applies an axial load to the elbow and a downward and
posterior force to the upper arm. A positive result causes a sudden onset of posterior shoulder pain. A
positive jerk test combined with a positive Kim test has a 97% sensitivity for posterior instability. After
extensive PT, the patient continues to have examination findings consistent with posterior shoulder
injury or instability, so an MRI scan or MRI arthrography would be helpful to assess for any pathology.
A subacromial injection is not indicated by this examination, which shows a strong rotator cuff and no
demonstrated bursal-sided symptoms. A CT scan can be helpful in scenarios involving bony pathology,
but an MRI is indicated at this stage in the evaluation of soft tissue. Although continuing PT may help
to abate symptoms, the patient was continuing to have symptoms with PT. The MRI arthrogram shows a
complex posterior labrum tear at the inferior to mid glenoid with separation of labrum from the glenoid.
Because the examination findings are consistent with the MRI findings and nonsurgical treatment has
failed to resolve symptoms, the next step is to recommend surgical treatment with labrum debridement
and/or repair. No examination or MRI findings indicate a need for injection, rotator cuff repair, ORIF, or
chondroplasty unless incidental intrasurgical findings are found.

2013 American Academy of Orthopaedic Surgeons 2013 Sports Medicine Self-Assessment Examination
2013 Sports Medicine Self-Assessment Examination Answer Book 41

RECOMMENDED READINGS FOR QUESTIONS 37 AND 38


Provencher MT, LeClere LE, King S, McDonald LS, Frank RM, Mologne TS, Ghodadra NS, Romeo AA.
Posterior instability of the shoulder: diagnosis and management. Am J Sports Med. 2011 Apr;39(4):874-
86. Epub 2010 Dec 4. PubMed PMID: 21131678.
Kim SH, Park JS, Jeong WK, Shin SK. The Kim test: a novel test for posteroinferior labral lesion of the
shoulder--a comparison to the jerk test. Am J Sports Med. 2005 Aug;33(8):1188-92. Epub 2005 Jul 6.
PubMed PMID: 16000664.
Vidal L, Bradley J. Management of posterior shoulder instability in the athlete. Curr Opin Orthop
2006;17:164-171.

END OF SERIES

Question 39
A high school athlete sustained a noncontact injury to his right knee. He says that during a football game
he felt a pop and his leg gave way. He attempted to continue to play but was unable secondary to pain.
Five days after the injury, radiographs of his right knee do not reveal any abnormalities. On examination,
he has an effusion on the injured side and no joint line tenderness. His range of motion is full extension
to 110 degrees of flexion. At 20 degrees of flexion, he has increased anterior translation compared to the
contralateral, uninjured left side. At 90 degrees of flexion, the tibia does not translate posteriorly. As his
knee is moved from full extension into flexion with an internal rotation and valgus force, you notice a
clunk within the knee. What is the most likely biomechanical basis for the clunk?

1. In extension, the medial tibial plateau is subluxated; as an internal rotation/valgus force is


applied in conjunction with flexion, the medial tibial plateau reduces.
2. In extension, the medial tibial plateau is reduced; as an internal rotation/valgus force is applied
in conjunction with flexion, the medial tibial plateau subluxates.
3. In extension, the lateral tibial plateau is reduced; as an internal rotation/valgus force is applied
in conjunction with flexion, the lateral plateau subluxates.
4. In extension, the lateral tibial plateau is subluxated; as an internal rotation/valgus force is
applied in conjunction with flexion, the lateral plateau reduces.

PREFERRED RESPONSE: 4

2013 American Academy of Orthopaedic Surgeons 2013 Sports Medicine Self-Assessment Examination
42 American Academy of Orthopaedic Surgeons

DISCUSSION
This patient sustained an isolated anterior cruciate ligament (ACL) injury based upon the mechanism
described and examination findings. Increased anterior translation at 20 degrees of flexion (a positive
Lachman test result) indicates an incompetent ACL. His other examination findings do not indicate a
torn posterior cruciate ligament or torn menisci. The examination finding that produces the clunk has
been termed the pivot shift maneuver and is positive in a knee with an incompetent ACL. Much has been
written about the pivot shift examination and the pathologic motions that occur during this test. With an
ACL-deficient knee in full extension and internal rotation, the lateral tibial plateau subluxates anteriorly.
When a valgus load is applied to the knee, the lateral plateau impinges on the lateral femoral condyle. As
the knee is flexed, the lateral tibial plateau slides posteriorly into a reduced position, causing an audible
clunk. Response 4 correctly describes the pathomechanics that result in the audible clunk heard during
the pivot shift maneuver. Responses 1 and 2 are incorrect because they describe the medial tibial plateau,
which is not part of the pathomechanics of the pivot shift. Response 3 is incorrect because in extension,
the lateral tibial plateau is subluxated, not reduced.

RECOMMENDED READINGS
Lane CG, Warren R, Pearle AD. The pivot shift. J Am Acad Orthop Surg. 2008 Dec;16(12):679-88.
Review. PubMed PMID: 19056917.
Galway HR, MacIntosh DL. The lateral pivot shift: a symptom and sign of anterior cruciate ligament
insufficiency. Clin Orthop Relat Res. 1980 Mar-Apr;(147):45-50. PubMed PMID: 7371314.

CLINICAL SITUATION FOR QUESTIONS 40 THROUGH 42


A 9-year-old boy was injured while playing soccer. His examination revealed painful range of motion
between 5 degrees and 75 degrees. There is tenderness on the medial side of his knee. There is no
effusion, a grade 1A Lachman, and severe pain over the medial epicondyle of the knee. Varus stress is
negative and pain is elicited with valgus stress. Initial radiographs were negative for abnormality.

Question 40
What is the next diagnostic step?

1. Repeat radiographs while the patient is weight bearing


2. Ultrasound of the lower extremity and calf
3. Stress radiographs
4. CT scan

PREFERRED RESPONSE: 3

2013 American Academy of Orthopaedic Surgeons 2013 Sports Medicine Self-Assessment Examination
2013 Sports Medicine Self-Assessment Examination Answer Book 43

Question 41
Stress radiographs show a 2-mm medial physeal widening with valgus stress. What is the best initial
treatment strategy for this patient?

1. Femoral medial collateral ligament repair, extraphyseal


2. Arthroscopically assisted medial collateral ligament repair
3. Crutch ambulation without immobilization and weight bearing as tolerated
4. Protected weight bearing with cast immobilization

PREFERRED RESPONSE: 4

Question 42
What is the most likely area of injury?

1. Femoral attachment of the medial collateral ligament


2. Tibial attachment of the medial collateral ligament
3. Hypertrophic zone of the growth plate
4. Proliferative zone of the growth plate

PREFERRED RESPONSE: 3

DISCUSSION FOR QUESTIONS 40 THROUGH 42


This patient likely has a physeal injury to the distal femoral physis. Stress radiographs or an MRI scan
will most reliably reveal this diagnosis. The growth plate, when injured, is most commonly fractured
through the hypertrophic zone of cartilage, its weakest point. This patient is optimally treated in a
cylindrical or long-leg cast. Younger patients can be treated with a hip spica with a leg extension.

RECOMMENDED READINGS FOR QUESTIONS 40 THROUGH 42


Zionts LE. Fractures around the knee in children. J Am Acad Orthop Surg. 2002 Sep-Oct;10(5):345-55.
Review. PubMed PMID: 12374485.
Wheeless C. Wheeless Textbook of Orthopaedics. Growth Plate Anatomy.
http://www.wheelessonline.com/ortho/growth_plate_anatomy Accessed 10/01/2012.

END OF SERIES

2013 American Academy of Orthopaedic Surgeons 2013 Sports Medicine Self-Assessment Examination
44 American Academy of Orthopaedic Surgeons

Question 43
An otherwise healthy 15-year-old wrestler has a 6-cm cutaneous lesion on the posterior aspect of his right
elbow that he reports as a spider bite. What is the most likely diagnosis?

1. Psoriasis
2. Tinea corporis
3. Herpes simplex virus
4. Community-acquired methicillin-resistant Staphylococcus aureus (CA-MRSA)

PREFERRED RESPONSE: 4

DISCUSSION
Patients who have skin and soft-tissue infections caused by CA-MRSA often describe the lesion as a
spider bite. The cytotoxin Panton-Valentine leukocidin that is produced by many strains of CA-MRSA
causes tissue necrosis, resulting in rapid development of an abscess and the appearance of a spider bite.
Patients with psoriasis have thick, red skin with flaky, silver-white patches. Tinea corporis is a general
term for a cutaneous fungal infection. The lesion appears as a well-demarcated erythematous plaque
with a raised border and central hypopigmentation, giving it a ring-like appearance. Primary infection
with herpes simplex virus can produce constitutional symptoms with burning, tingling, or stinging at the
site. Grouped vesicles with clear fluid 1 to 2 mm in size form on an erythematous base and then rupture,
leaving moist ulcers or crusted plaques.

RECOMMENDED READINGS
Rihn JA, Michaels MG, Harner CD. Community-acquired methicillin-resistant staphylococcus aureus:
an emerging problem in the athletic population. Am J Sports Med. 2005 Dec;33(12):1924-9. Review.
PubMed PMID: 16314668.
Marcotte AL, Trzeciak MA. Community-acquired methicillin-resistant Staphylococcus aureus: an
emerging pathogen in orthopaedics. J Am Acad Orthop Surg. 2008 Feb;16(2):98-106. Review. PubMed
PMID: 18252840.

2013 American Academy of Orthopaedic Surgeons 2013 Sports Medicine Self-Assessment Examination
2013 Sports Medicine Self-Assessment Examination Answer Book 45

Figure 44

CLINICAL SITUATION FOR QUESTIONS 44 AND 45


Figure 44 is the MRI scan of a 14-year-old soccer player who injured his right knee during a game.
He describes feeling a pop and he needed help walking off the field. His knee is visibly swollen. A
Lachman test demonstrates asymmetry with no endpoint.

Question 44
Range of motion of the knee is between 0 degrees and 70 degrees. What is the most appropriate treatment
option?

1. Nonweight-bearing activity with crutches


2. Microfracture of the chondral defect
3. Immediate anterior cruciate ligament (ACL) reconstruction
4. Delayed ACL reconstruction

PREFERRED RESPONSE: 4

Question 45
The patient has no postsurgical complications and begins physical therapy rehabilitation. The boy and his
parents stress they want to get the therapy over with as fast as possible to expedite his return to sports,
and the surgeon and rehabilitation team consider their request. Compared to nonaccelerated rehabilitation,
patients who follow an early accelerated rehabilitation protocol experience

1. increased laxity.
2. no differences in long-term results.
3. increased risk for graft failure.
4. lower Knee Injury and Osteoarthritis Outcome Scores (KOOS).

PREFERRED RESPONSE: 2

2013 American Academy of Orthopaedic Surgeons 2013 Sports Medicine Self-Assessment Examination
46 American Academy of Orthopaedic Surgeons

DISCUSSION FOR QUESTIONS 44 AND 45


The Lachman test is the most sensitive examination for acute ACL injuries. ACL injury rates are higher
in women than in men. This likely is attributable to anatomic differences (smaller notches, smaller
ligaments, and different landing biomechanics). Lateral meniscal tears are more common than medial
tears. The anterior drawer test is accentuated with 30 degrees of external rotation of the tibia. The
MRI scan shows bone bruises consistent with an ACL tear. It is also always necessary to examine other
structures of the knee in patients with ACL tears. All ligaments of the affected knee should be assessed.
Lateral meniscal tears are frequently associated with ACL tears. Medial meniscal tears and posterolateral
corner deficiency are also possible. Randomized clinical trials comparing an early accelerated vs
nonaccelerated rehabilitation have demonstrated no significant differences in long-term results. These
studies did not address timing of return to play with an early accelerated rehabilitation program. At 2 and
3 years postsurgically, there are no differences in laxity, number of graft failures, or KOOS scores.

RECOMMENDED READINGS FOR QUESTIONS 44 AND 45


Gurtler RA, Stine R, Torg JS. Lachman test evaluated. Quantification of a clinical observation. Clin
Orthop Relat Res. 1987 Mar;(216):141-50. PubMed PMID: 3815941.
Safran MC, Soto G. Meniscus: diagnosis and decision making. In Miller MD, Cole BJ, eds. Textbook of
Arthroscopy. Philadelphia, PA: WB Saunders; 2004:502.
Shelbourne KD, Nitz PA. The O'Donoghue triad revisited. Combined knee injuries involving anterior
cruciate and medial collateral ligament tears. Am J Sports Med. 1991 Sep-Oct;19(5):474-7. PubMed
PMID: 1962712.
Bales CP, Guettler JH, Moorman CT 3rd. Anterior cruciate ligament injuries in children with open physes:
evolving strategies of treatment. Am J Sports Med. 2004 Dec;32(8):1978-85. Review. PubMed PMID:
15572331.
McCarroll JR, Rettig AC, Shelbourne KD. Anterior cruciate ligament injuries in the young athlete with
open physes. Am J Sports Med. 1988 Jan-Feb;16(1):44-7. Review. PubMed PMID: 3278634.
Beynnon BD, Johnson RJ, Naud S, Fleming BC, Abate JA, Brattbakk B, Nichols CE. Accelerated versus
nonaccelerated rehabilitation after anterior cruciate ligament reconstruction: a prospective, randomized,
double-blind investigation evaluating knee joint laxity using roentgen stereophotogrammetric analysis.
Am J Sports Med. 2011 Dec;39(12):2536-48. Epub 2011 Sep 27. PubMed PMID: 21952714.
Glasgow SG, Gabriel JP, Sapega AA, Glasgow MT, Torg JS. The effect of early versus late return to
vigorous activities on the outcome of anterior cruciate ligament reconstruction. Am J Sports Med. 1993
Mar-Apr;21(2):243-8. PubMed PMID: 8465920.

END OF SERIES

2013 American Academy of Orthopaedic Surgeons 2013 Sports Medicine Self-Assessment Examination
2013 Sports Medicine Self-Assessment Examination Answer Book 47

Question 46
Chronic traumatic encephalopathy (CTE) is a neurodegenerative disease that is characterized by

1. onset most often by age 30.


2. a temporary state of neuronal and axonal derangement.
3. manifestations of affect such as apathy, irritability, and suicidal ideation.
4. absence of gross pathological brain changes upon autopsy.

PREFERRED RESPONSE: 3

DISCUSSION
CTE is a neurodegenerative disease that occurs years or decades after recovery from acute or postacute
effects of head trauma. The exact relationship between concussion and CTE is not entirely clear; however,
early behavioral manifestations of CTE have been described by family and providers to include apathy,
irritability, and suicidal ideation. For some patients, cognitive difficulty such as poor episodic memory
and executive function may be the first signs of CTE. Onset most often occurs in midlife after athletes
have completed their sports careers, with mean age of onset at 42 years. The effects on the brain are
degenerative, leading to a permanent state of derangement. Autopsy findings demonstrate multiple gross
pathological findings. The condition is more common among contact athletes.

RECOMMENDED READINGS
Gavett BE, Stern RA, McKee AC. Chronic traumatic encephalopathy: a potential late effect of sport-
related concussive and subconcussive head trauma. Clin Sports Med. 2011 Jan;30(1):179-88, xi. Review.
PubMed PMID: 21074091.
McKee AC, Cantu RC, Nowinski CJ, Hedley-Whyte ET, Gavett BE, Budson AE, Santini VE, Lee
HS, Kubilus CA, Stern RA. Chronic traumatic encephalopathy in athletes: progressive tauopathy after
repetitive head injury. J Neuropathol Exp Neurol. 2009 Jul;68(7):709-35. Review. PubMed PMID:
19535999; PubMed Central PMCID: PMC2945234.

Question 47
In the shoulder position of 90-degree forward flexion and internal rotation, what is the most important
static stabilizer of the glenohumeral joint?

1. Rotator interval
2. Infraspinatus
3. Anterior band of the inferior glenohumeral ligament
4. Posterior band of the inferior glenohumeral ligament

PREFERRED RESPONSE: 4

2013 American Academy of Orthopaedic Surgeons 2013 Sports Medicine Self-Assessment Examination
48 American Academy of Orthopaedic Surgeons

DISCUSSION
In the position of 90 degrees forward flexion and internal rotation, the most important static stabilizer of
the glenohumeral joint is the posterior band of the inferior glenohumeral ligament. This position places
the posterior-inferior glenohumeral ligament in an anterior-posterior direction and under tension. The
superior glenohumeral ligament and the middle glenohumeral ligament provide static stability in the fully
adducted and midrange-adducted positions, respectively. The subscapularis and infraspinatus provide
primarily dynamic stability to the glenohumeral joint. Though not fully clear, the rotator interval appears
to provide more static stability with the arm adducted, limiting inferior and posterior translation, and less
so in the forward flexion and internal rotation position.

RECOMMENDED READINGS
Blasier RB, Soslowsky LJ, Malicky DM, Palmer ML. Posterior glenohumeral subluxation: active and
passive stabilization in a biomechanical model. J Bone Joint Surg Am. 1997 Mar;79(3):433-40. PubMed
PMID: 9070535.
Provencher MT, LeClere LE, King S, McDonald LS, Frank RM, Mologne TS, Ghodadra NS, Romeo AA.
Posterior instability of the shoulder: diagnosis and management. Am J Sports Med. 2011 Apr;39(4):874-
86. Epub 2010 Dec 4. PubMed PMID: 21131678.
Matsen FA 3rd, Chebli C, Lippitt S; American Academy of Orthopaedic Surgeons. Principles for the
evaluation and management of shoulder instability. J Bone Joint Surg Am. 2006 Mar;88(3):648-59.
PubMed PMID: 16541527.

CLINICAL SITUATION FOR QUESTIONS 48 AND 49


A 17-year-old high school wrestler sustains an abrasion over the posterior aspect of his right elbow during
a match. During the next few days the abrasion becomes erythematous and he is placed on oral cephalexin
500 mg four times per day. The erythema extends proximally despite the antibiotic regimen. The patient
is afebrile, there is no fluid collection associated with the lesion, and his elbow joint is not involved.

Question 48
What is the most appropriate treatment?

1. Switch to oral trimethoprim-sulfamethoxazole double-strength twice per day for 10 to 14 days


2. Switch to oral ciprofloxacin 500 mg twice per day for 10 to 14 days
3. Begin cefazolin 1 gram intravenously (IV) every 8 hours for 7 to 10 days
4. Irrigation and debridement with empiric IV antibiotic coverage

PREFERRED RESPONSE: 1

2013 American Academy of Orthopaedic Surgeons 2013 Sports Medicine Self-Assessment Examination
2013 Sports Medicine Self-Assessment Examination Answer Book 49

Question 49
Assuming that the lesion can be covered appropriately and there is no drainage from the lesion, when
should the patient be allowed to safely return to wrestling?

1. When the absence of pain is reported by the wrestler for 3 consecutive days
2. When 72 hours of antibiotics have been administered and there is no extension of the lesion
for 48 hours
3. When laboratory values are within defined limits and the patient remains afebrile for 3 days
4. When the lesion has decreased in size by 50%

PREFERRED RESPONSE: 2

DISCUSSION FOR QUESTIONS 48 AND 49


This patient has cellulitis, which is typically caused by group A Streptococcus or Staphylococcus.
The patients lack of improvement with first-line antibiotics is concerning for methicillin-resistant
Staphylococcus aureus (MRSA) infection. MRSA cellulitis is becoming more prevalent in young athletes,
and a high index of suspicion is required to provide appropriate intervention during this aggressive disease
process. The diagnosis is typically made clinically without the use of cultures. Oral trimethoprim-
sulfamethoxazole (a sulfonamide-class drug) double strength twice daily for 10 to 14 days or doxycycline
(a tetracycline-class drug) 100 mg twice daily for 10 to 14 days are recommended for first-line treatment
of suspected MRSA cellulitis. There is no indication to proceed with irrigation and debridement; however,
if the patient develops a soft-tissue abscess or the underlying joint becomes involved, this would be an
appropriate intervention. Switching the athlete to an IV cephalosporin (cefazolin) is not likely to be
effective against the presumed resistant bacteria. Ciprofloxacin (a fluoroquinolone-class drug) is effective
against many bacteria, but not MRSA. The current recommendation for wrestlers with cellulitis is that
return to competition be allowed after 72 hours of antibiotic treatment if there has been no extension of
the cellulitis for 48 hours, the lesion can be covered, and there is no drainage from the lesion. The other
responses are not current recommendations for return to competition.

RECOMMENDED READINGS FOR QUESTIONS 48 AND 49


Fields KB, Rowand M, Samuhel K. Infectious disease in athletes. In: Kibler WB, ed. Orthopaedic
Knowledge Update: Sports Medicine 4. Rosemont, IL: American Academy of Orthopaedic Surgeons;
2009:365-378.
Liu C, Bayer A, Cosgrove SE, Daum RS, Fridkin SK, Gorwitz RJ, Kaplan SL, Karchmer AW, Levine DP,
Murray BE, J Rybak M, Talan DA, Chambers HF. Clinical practice guidelines by the infectious diseases
society of America for the treatment of methicillin-resistant Staphylococcus aureus infections in adults and
children: executive summary. Clin Infect Dis. 2011 Feb 1;52(3):285-92. PubMed PMID: 21217178.
NCAA Wrestling Committee: NCAA Wrestling: 2008 Mens Rules and Interpretations. Indianapolis, IN:
2007:1-182.

END OF SERIES

2013 American Academy of Orthopaedic Surgeons 2013 Sports Medicine Self-Assessment Examination
50 American Academy of Orthopaedic Surgeons

Question 50
A 19-year-old linebacker underwent a coracoid transfer procedure for recurrent anterior glenohumeral
instability. At his 1-week postsurgical check-up, his incision is doing well; however, he reports numbness
over the lateral aspect of his forearm. What nerve may have been injured during his surgery?

1. Axillary
2. Median
3. Musculocutaneous
4. Radial

PREFERRED RESPONSE: 3

DISCUSSION
This patient has sustained an injury to the musculocutaneous nerve. The terminal branch of this nerve is
the lateral antebrachial cutaneous nerve of the forearm. The axillary nerve provides sensation to the lateral
arm. The median nerve provides sensation more distally. The radial nerve is not likely to be injured with
a coracoid transfer procedure; if it is, the injury would result in numbness near the wrist.

RECOMMENDED READINGS
Young AA, Maia R, Berhouet J, Walch G. Open Latarjet procedure for management of bone loss in
anterior instability of the glenohumeral joint. J Shoulder Elbow Surg. 2011 Mar;20(2 Suppl):S61-9. Epub
2010 Dec 9. Review. PubMed PMID: 21145262.
Richards RR, Hudson AR, Bertoia JT, Urbaniak JR, Waddell JP. Injury to the brachial plexus during
Putti-Platt and Bristow procedures. A report of eight cases. Am J Sports Med. 1987 Jul-Aug;15(4):374-80.
PubMed PMID: 3661820.

Question 51
What is the most important genetic element that distinguishes community-acquired methicillin-resistant
Staphylococcus aureus (CA-MRSA) from hospital-acquired MRSA?

1. Beta-lactamase
2. Penicillin-binding protein 2a
3. Panton-Valentine leukocidin (PVL)
4. Staphylococcus cassette chromosome (SCCmec) type I

PREFERRED RESPONSE: 3

2013 American Academy of Orthopaedic Surgeons 2013 Sports Medicine Self-Assessment Examination
2013 Sports Medicine Self-Assessment Examination Answer Book 51

DISCUSSION
PVL is a cytotoxin that defines CA-MRSA and is not typical of hospital-acquired MRSA. PVL has
the ability to lyse white blood cells and cause tissue necrosis, allowing for rapid progression of abscess
formation. Beta-lactamase is an enzyme that breaks the beta-lactam bond of penicillin and is present in
most strains of Staphylococcus aureus today. Synthetic penicillins such as methicillin are resistant to
the effects of beta-lactamase. MRSA and CA-MRSA carry the mecA gene, which encodes a penicillin-
binding protein with a very low affinity for beta-lactam antibiotics, resulting in methicillin resistance.
SCCmec mobile genetic units carry the mecA gene with additional genetic elements that together yield the
multidrug-resistant strains found in healthcare environments. SCCmec type IV is specific to CA-MRSA
and lacks these additional genetic elements, resulting in less multidrug resistance.

RECOMMENDED READINGS
Rihn JA, Michaels MG, Harner CD. Community-acquired methicillin-resistant staphylococcus aureus:
an emerging problem in the athletic population. Am J Sports Med. 2005 Dec;33(12):1924-9. Review.
PubMed PMID: 16314668.
Marcotte AL, Trzeciak MA. Community-acquired methicillin-resistant Staphylococcus aureus: an
emerging pathogen in orthopaedics. J Am Acad Orthop Surg. 2008 Feb;16(2):98-106. Review. PubMed
PMID: 18252840.

Figure 52a Figure 52b Figure 52c Figure 52d

CLINICAL SITUATION FOR QUESTIONS 52 THROUGH 54


A 13-year-old baseball player fell while rounding second base 2 days ago. He said it felt like his knee
buckled when he turned toward third base. He could not finish the game, but was able to bear weight
with a limp. He has had no previous knee injuries, but now complains of pain in his right knee. Initial
examination demonstrated an effusion, tenderness at the proximal medial collateral region and medial
patellofemoral retinaculum, decreased range of motion, and patella apprehension. He was otherwise
ligamentously stable. No other noteworthy physical findings were found.

2013 American Academy of Orthopaedic Surgeons 2013 Sports Medicine Self-Assessment Examination
52 American Academy of Orthopaedic Surgeons

Question 52
What is the next treatment step?

1. Radiographs
2. Arthroscopy
3. MRI scan
4. Duplex ultrasound

PREFERRED RESPONSE: 1

Question 53
What do the radiographs shown in Figures 52a and 52b reveal?

1. Medial femoral chondyle physeal widening


2. Osseous or osteochondral loose fragment
3. Osgood-Schlatter disease
4. Patella nondisplaced fracture

PREFERRED RESPONSE: 2

Question 54
Figures 52c and 52d show the proton density fat-saturated MRI scans. Treatment at this stage includes
arthroscopy and

1. early functional rehabilitation.


2. proximal realignment alone.
3. attempted internal fixation.
4. medial collateral ligament (MCL) repair.

PREFERRED RESPONSE: 3

2013 American Academy of Orthopaedic Surgeons 2013 Sports Medicine Self-Assessment Examination
2013 Sports Medicine Self-Assessment Examination Answer Book 53

DISCUSSION FOR QUESTIONS 52 THROUGH 54


This patients examination indicates a patellar or peripatellar knee injury. Initial evaluation with
radiographs will assess for fracture, subluxation, or osteochondral injury. Examination findings did not
demonstrate a need for emergent surgery, an MRI scan, or an ultrasound, so radiographs are the initial
diagnostic imaging choice. Radiographs show an osseous or osteochondral loose fragment. There is
no evidence of obvious nondisplaced fracture or physeal changes. In suspected patella dislocation or
subluxation with loose fragment seen on radiographs, an MRI scan is indicated. Lateral release alone
is seldom indicated in a knee that was normal before injury. Acute proximal realignment has not been
shown to alter long-term outcomes for first-time dislocators. The examination and MRI scan did not
indicate a need for MCL repair. Closed reduction of the osteochondral fragment would not be indicated or
appropriate for this injury. Treatment should consist of arthroscopy or arthrotomy and attempted internal
fixation of this fragment. If fixation is not possible, the loose body can be removed.

RECOMMENDED READINGS FOR QUESTIONS 52 THROUGH 54


Frank JB, Jarit GJ, Bravman JT, Rosen JE. Lower extremity injuries in the skeletally immature athlete. J
Am Acad Orthop Surg. 2007 Jun;15(6):356-66. Review. PubMed PMID: 17548885.
Fithian DC, Paxton EW, Stone ML, Silva P, Davis DK, Elias DA, White LM. Epidemiology and natural
history of acute patellar dislocation. Am J Sports Med. 2004 Jul-Aug;32(5):1114-21. Epub 2004 May 18.
PubMed PMID: 15262631.
Iobst CA, Stanitski CL. Acute knee injuries. Clin Sports Med. 2000 Oct;19(4):621-35, vi. Review.
PubMed PMID: 11019732.

END OF SERIES

Question 55
While obtaining informed consent for a lateral closing-wedge osteotomy, what complication should be
discussed with the patient as exclusive to this procedure and not encountered in medial opening-wedge
osteotomy?

1. Compartment syndrome
2. Plate breakage
3. Neurologic injury
4. Proximal tibiofibular joint disruption

PREFERRED RESPONSE: 4

DISCUSSION
With lateral closing-wedge osteotomy, proximal tibiofibular disruption can occur. This is not seen in
medial opening-wedge osteotomy. A technique has been developed to prevent this complication; a fibular
osteotomy is performed at the same time as the tibial osteotomy. The other complications listed are
seen in both techniques, with nonunion and plate breakage more common in opening-wedge high tibial
osteotomy (HTO) and neurologic injury more common in closing-wedge HTO (with issues related to the
common peroneal nerve most prevalent). Compartment syndrome is a devastating complication that can
occur with any osteotomy, and a high index of suspicion should be maintained during the postsurgical
course for patients who develop this condition.

2013 American Academy of Orthopaedic Surgeons 2013 Sports Medicine Self-Assessment Examination
54 American Academy of Orthopaedic Surgeons

RECOMMENDED READINGS
Amendola A, Bonasia DE. Results of high tibial osteotomy: review of the literature. Int Orthop. 2010
Feb;34(2):155-60. Epub 2009 Oct 17. Review. PubMed PMID: 19838706.
Rossi R, Bonasia DE, Amendola A. The role of high tibial osteotomy in the varus knee. J Am Acad Orthop
Surg. 2011 Oct;19(10):590-9. Review. PubMed PMID: 21980024.

Figure 56

CLINICAL SITUATION FOR QUESTIONS 56 THROUGH 58


Figure 56 is the MRI scan of a 15-year-old girl who had left knee pain after sustaining a noncontact
twisting injury while playing soccer. She reported severe pain initially that has since improved. On
examination, she had a large knee effusion with lateral joint line tenderness. Range of motion is from
5 degrees of extension to 70 degrees of flexion. She wishes to return to sports at her preinjury level of
activity.

Question 56
What examination test is most likely to reveal abnormal findings?

1. Pivot shift test


2. Quadriceps active test
3. Patellar apprehension test
4. External rotation recurvatum test

PREFERRED RESPONSE: 1

2013 American Academy of Orthopaedic Surgeons 2013 Sports Medicine Self-Assessment Examination
2013 Sports Medicine Self-Assessment Examination Answer Book 55

Question 57
What is the most appropriate treatment?

1. Delayed ligament reconstruction


2. Physical therapy and functional bracing
3. Immediate ligament repair
4. Immediate ligament reconstruction

PREFERRED RESPONSE: 1

Question 58
What is the most likely mechanism of injury?

1. External rotation
2. Posterior translation
3. Hyperextension and varus
4. Anterior translation and internal rotation

PREFERRED RESPONSE: 4

DISCUSSION FOR QUESTIONS 56 THROUGH 58


The MRI scan shows a bone bruise of the lateral femoral condyle and lateral tibial plateau. This injury
pattern is commonly associated with anterior cruciate ligament (ACL) rupture and an abnormal pivot shift
test result. Treatment of an ACL tear in a high-demand athlete should consist of ligament reconstruction.
In this patient, surgery should be delayed until she regains full range of motion to minimize risk for
arthrofibrosis after surgery. Recent analysis has shown that the noncontact mechanism is more consistent
with anterior translation, affecting both the medial and lateral compartments. The bone bruise in the
lateral femoral condyle occurs more anterior than that of the medial femoral condyle, suggesting that
internal rotation has occurred. The external rotation recurvatum test assesses for posterolateral corner
injury, and a positive quadriceps active test is consistent with posterior cruciate ligament rupture. An
abnormal patellar apprehension test result is suggestive of patellar instability. Nonsurgical treatment is
unlikely to result in sufficient stability if this patient returns to sports at her preinjury level of activity.
Primary ACL repair is associated with high failure rates. Although the precise mechanism of injury varies,
injuries can be broadly classified into contact and noncontact injuries. Noncontact injuries occur with
the knee in slight flexion, valgus, and internal rotation, and contact injuries typically involve a lateral-
side impact producing a valgus force to the knee. The valgus component of noncontact injuries has been
thought to cause mainly lateral compartment bone bruising. Posterior translation is the most common
mechanism of posterior cruciate ligament rupture, and hyperextension and varus is associated with
posterolateral corner injury.

2013 American Academy of Orthopaedic Surgeons 2013 Sports Medicine Self-Assessment Examination
56 American Academy of Orthopaedic Surgeons

RECOMMENDED READINGS FOR QUESTIONS 56 THROUGH 58


Kaar SG, Stuart MJ, Levy BA. Soft-tissue injuries about the knee. In: Flynn JM, ed. Orthopaedic
Knowledge Update 10. Rosemont, IL: American Academy of Orthopaedic Surgeons; 2011:453-467.
Viskontas DG, Giuffre BM, Duggal N, Graham D, Parker D, Coolican M. Bone bruises associated with
ACL rupture: correlation with injury mechanism. Am J Sports Med. 2008 May;36(5):927-33. Epub 2008
Mar 19. PubMed PMID: 18354139.

END OF SERIES

Question 59
A 12-year-old boy who plays multiple sports has had insidious-onset heel pain while running for 4
months. On examination, he had ankle dorsiflexion of 5 degrees. The squeeze test result was positive
and the Thompson test result was negative. He has no pain with forced ankle plantar flexion. What is the
most likely diagnosis?

1. Achilles rupture
2. Gastrocnemius strain
3. Calcaneal apophysitis
4. Os trigonum syndrome

PREFERRED RESPONSE: 3

DISCUSSION
Calcaneal apophysitis (Sever disease) is a common cause of heel pain in adolescent athletes who
participate in running or jumping sports. The condition occurs primarily before or during peak growth
and is characterized by a tight Achilles tendon, a positive squeeze test, and tenderness over the calcaneal
apophysis. Pain is localized to the heel and exacerbated by running. Os trigonum syndrome involves
posterior ankle impingement and is commonly associated with ballet dancers. Gastrocnemius strain
typically causes pain more proximally at the myotendinous junction. The Thompson test is performed
with the patient lying prone on the examination table. Absence of ankle plantar flexion when the examiner
squeezes the calf constitutes a positive test and is indicative of Achilles rupture.

RECOMMENDED READINGS
Shea K, Ganley TJ. Injuries and conditions of the pediatric and adolescent athlete. In: Flynn JM, ed.
Orthopaedic Knowledge Update 10. Rosemont, IL: American Academy of Orthopaedic Surgeons;
2011:789-796.
Frank JB, Jarit GJ, Bravman JT, Rosen JE. Lower extremity injuries in the skeletally immature athlete. J
Am Acad Orthop Surg. 2007 Jun;15(6):356-66. Review. PubMed PMID: 17548885.

2013 American Academy of Orthopaedic Surgeons 2013 Sports Medicine Self-Assessment Examination
2013 Sports Medicine Self-Assessment Examination Answer Book 57

CLINICAL SITUATION FOR QUESTIONS 60 AND 61


A 15-year-old boy has had shoulder pain for 4 weeks during the middle of baseball season. The patient
says his pain is all over my shoulder. Examination reveals tenderness to palpation over the anterolateral
aspect of the shoulder. Internal and external rotation range of motion is restricted. Radiographs show
metaphyseal sclerosis.

Question 60
What is the most significant risk factor for the development of little leaguers shoulder in this scenario?

1. Recent increase in the number of pitches


2. Gender (male)
3. Glenohumeral internal rotation deficit
4. Genetic factors

PREFERRED RESPONSE: 1

Question 61
This boys parents are eager to get him back on the field as soon as possible. What is the most appropriate
treatment option?

1. Screw fixation of the epiphysis


2. Arthroscopic debridement
3. A shut-down period until the boy is asymptomatic, and gradual return to pitching via a
throwing program
4. An intra-articular cortisone injection

PREFERRED RESPONSE: 3

DISCUSSION FOR QUESTIONS 60 AND 61


Although a recent increase in the number of pitches may have contributed to this patients development of
little leaguers shoulder, the most significant overall factor is age. Little leaguers shoulder is caused by
rotational stress placed on the proximal humeral epiphysis during overhead throwing. The growth plate
is weakest to torsion stress, and is most susceptible to injury during periods of rapid growth commonly
seen during puberty. Most chronic shoulder injuries occur in throwing athletes between 13 and 16 years
of age. Genetic factors and gender have not been studied in association with little leaguers shoulder. An
initial 3-month period of rest and activity modification will typically result in resolution of symptoms.
Nonsteroidal anti-inflammatory drugs may be used as needed. After the rest period, a gradual return
to baseline pitching is implemented until the patient is back to baseline. This protocol has a long-term
success rate exceeding 90%.

2013 American Academy of Orthopaedic Surgeons 2013 Sports Medicine Self-Assessment Examination
58 American Academy of Orthopaedic Surgeons

RECOMMENDED READINGS FOR QUESTIONS 60 AND 61


Leonard J, Hutchinson MR. Shoulder injuries in skeletally immature throwers: review and current
thoughts. Br J Sports Med. 2010 Apr;44(5):306-10. Review. PubMed PMID: 20371555.
Osbahr DC, Kim HJ, Dugas JR. Little league shoulder. Curr Opin Pediatr. 2010 Feb;22(1):35-40. Review.
PubMed PMID: 19926993.

END OF SERIES

Figure 62

Question 62
Figure 62 is an arthroscopic view of the intercondylar notch of a right knee from an anterolateral portal.
What is the main function of the structure delineated by the black asterisks?

1. Resist anterior translation during knee flexion


2. Resist posterior translation during knee flexion
3. Resist rotatory loads during knee flexion
4. Resist rotatory loads during knee extension

PREFERRED RESPONSE: 4

DISCUSSION
The structure shown is the posterolateral bundle of the anterior cruciate ligament (ACL). This bundle is
optimally positioned in the knee to resist rotatory forces during terminal knee extension. Resist anterior
translation during knee flexion best describes the anteromedial bundle. Resist rotatory loads during
knee flexion is unlikely because the posterolateral bundle is tightest during knee extension. The posterior
cruciate ligament, not the ACL, functions to resist posterior translation.

RECOMMENDED READINGS
Petersen W, Zantop T. Anatomy of the anterior cruciate ligament with regard to its two bundles. Clin
Orthop Relat Res. 2007 Jan;454:35-47. Review. PubMed PMID: 17075382.
Zantop T, Petersen W, Sekiya JK, Musahl V, Fu FH. Anterior cruciate ligament anatomy and function
relating to anatomical reconstruction. Knee Surg Sports Traumatol Arthrosc. 2006 Oct;14(10):982-92.
Epub 2006 Aug 5. Review. PubMed PMID: 16897068.

2013 American Academy of Orthopaedic Surgeons 2013 Sports Medicine Self-Assessment Examination
2013 Sports Medicine Self-Assessment Examination Answer Book 59

Question 63
A 25-year-old wrestler has been experiencing increasing left knee pain since his last professional cage
fight. He complains of both pain and instability on the medial side of his left knee. Examination reveals
a grade 3 Lachman and pseudolaxity with valgus stress. Dial test findings are normal. Radiographs show
medial degenerative changes and 5 degrees of varus alignment. What is the most appropriate treatment?

1. Rehabilitation with vibration-platform weight-bearing squats


2. Anterior cruciate ligament (ACL) reconstruction with autograft bone-tendon-bone
3. High-tibial osteotomy (HTO)
4. HTO plus ACL reconstruction at the same time

PREFERRED RESPONSE: 4

DISCUSSION
A young athlete with posttraumatic arthrosis of the knee isolated to the medial side poses a challenge.
History and examination confirm key findings. Complaints of both pain and instability warrant
concomitant HTO plus an ACL procedure, particularly in young athletes. Rehabilitation with vibration
platform is an unproven technique and is not specific to this knee problem. ACL reconstruction or HTO in
isolation would not be ideal for this young, active patient.

RECOMMENDED READINGS
Amendola A, Panarella L. High tibial osteotomy for the treatment of unicompartmental arthritis of the
knee. Orthop Clin North Am. 2005 Oct;36(4):497-504. Review. PubMed PMID: 16164954.
Voos JE, Suero EM, Citak M, Petrigliano FP, Bosscher MR, Citak M, Wickiewicz TL, Pearle AD. Effect
of tibial slope on the stability of the anterior cruciate ligament-deficient knee. Knee Surg Sports Traumatol
Arthrosc. 2012 Aug;20(8):1626-31. Epub 2011 Dec 20. PubMed PMID: 22183736.

Question 64
What is an example of cognitive rest after concussion?

1. Playing chess
2. Increasing reading
3. Working online
4. Limiting video games

PREFERRED RESPONSE: 4

DISCUSSION
Physical and cognitive rest are recommended as treatment for sports-related concussion. Cognitive rest
involves minimizing activities that require concentration and attention such as reading, schoolwork, video
games, text messaging, working online, and playing games that require concentration such as chess.
Limiting or eliminating video games after concussion is a form of cognitive rest.

2013 American Academy of Orthopaedic Surgeons 2013 Sports Medicine Self-Assessment Examination
60 American Academy of Orthopaedic Surgeons

RECOMMENDED READINGS
Meehan WP 3rd. Medical therapies for concussion. Clin Sports Med. 2011 Jan;30(1):115-24, ix. Review.
PubMed PMID: 21074086.
McCrory P, Meeuwisse W, Johnston K, Dvorak J, Aubry M, Molloy M, Cantu R. Consensus statement on
concussion in sport - the Third International Conference on Concussion in Sport held in Zurich, November
2008. Phys Sportsmed. 2009 Jun;37(2):141-59. Review. PubMed PMID: 20048521.

Figure 65

CLINICAL SITUATION FOR QUESTIONS 65 THROUGH 67


Figure 65 is the radiograph of a 24-year-old man who had left knee pain after sustaining a twisting injury
while playing Ultimate Frisbee. On examination, he had a large effusion with tenderness over the lateral
joint line. Pivot shift testing results were positive. Prone dial testing results at 30 degrees and 90 degrees
were negative. An MRI scan shows a tear of the anterior cruciate ligament (ACL).

Question 65
Based on the radiograph, the attachment for which structure has been disrupted?

1. Iliotibial band
2. Popliteus tendon
3. Lateral meniscus
4. Lateral capsular ligament

PREFERRED RESPONSE: 4

2013 American Academy of Orthopaedic Surgeons 2013 Sports Medicine Self-Assessment Examination
2013 Sports Medicine Self-Assessment Examination Answer Book 61

Question 66
Compared with a transtibial technique, what effect will drilling the femoral tunnel through the
anteromedial portal have?

1. Produces a longer femoral tunnel


2. Improves visualization while drilling
3. Should be performed at 90 degrees of knee flexion
4. Allows for independent access to the anatomic femoral insertion

PREFERRED RESPONSE: 4

Question 67
The patient requests anatomic double-bundle ACL reconstruction. Compared with transtibial single-
bundle ACL reconstruction, anatomic double-bundle ACL reconstruction is more likely to

1. restore improved knee kinematics.


2. allow for earlier return to sports.
3. result in better clinical outcomes scores.
4. be associated with lower surgery cost.

PREFERRED RESPONSE: 1

DISCUSSION FOR QUESTIONS 65 THROUGH 67


The radiograph shows a Segond fracture, an avulsion fracture involving the lateral capsular ligament.
This radiographic finding has been associated with ACL rupture in 75% to 100% of cases. Drilling
the femoral tunnel through the anteromedial portal allows for independent access to the native femoral
attachment. Fiber orientation is more oblique than with a transtibial technique and more closely resembles
that of the native ligament. Double-bundle reconstruction attempts to duplicate native ACL anatomy.
Biomechanical studies have shown that double-bundle reconstruction more closely reproduces normal
knee kinematics; however, this technique does not offer a clear advantage in terms of clinical outcomes.
The iliotibial band inserts onto Gerdys tubercle. The popliteus tendon originates from the lateral femoral
condyle. The lateral meniscus attaches near the intercondylar eminence at the anterior and posterior
meniscal roots. Recent advances in ACL reconstruction focus on restoring the native ACL anatomy.
Studies have determined that a knee flexion angle of 110 degrees is optimal to avoid blowout of the
back wall and injury to the lateral structures while drilling. Femoral tunnel length is typically shorter
than with a transtibial approach and decreases with higher-flexion angles. Double-bundle reconstruction
is associated with higher surgical costs because of the need for additional fixation and, in the case of
allograft reconstruction, a second graft.

2013 American Academy of Orthopaedic Surgeons 2013 Sports Medicine Self-Assessment Examination
62 American Academy of Orthopaedic Surgeons

RECOMMENDED READINGS FOR QUESTIONS 65 THROUGH 67


Kaar SG, Stuart MJ, Levy BA. Soft-tissue injuries about the knee. In: Flynn JM, ed. Orthopaedic
Knowledge Update 10. Rosemont, IL: American Academy of Orthopaedic Surgeons; 2011:453-467.
Seon JK, Gadikota HR, Wu JL, Sutton K, Gill TJ, Li G. Comparison of single-and double-bundle anterior
cruciate ligament reconstructions in restoration of knee kinematics and anterior cruciate ligament forces.
Am J Sports Med. 2010 Jul;38(7):1359-67. Epub 2010 Apr 14. PubMed PMID: 20392968.
Sanders TG, Miller MD. A systematic approach to magnetic resonance imaging interpretation of sports
medicine injuries of the knee. Am J Sports Med. 2005 Jan;33(1):131-48. Review. PubMed PMID:
15611010.

END OF SERIES

Question 68
A 42-year-old man has increasing pain and, to a lesser extent, some occasional left knee instability.
Several years earlier he sustained a noncontact twisting injury to his knee. He had some initial soreness
and pain but was able to resume his normal activities while avoiding sports. On examination, the patient
has medial joint line pain, a grade 2+ Lachman, and a slight varus thrust. His radiographs reveal mild-to-
moderate medial compartment osteoarthritis with varus alignment. What surgical treatment strategy likely
will alleviate his pain?

1. Distal femoral osteotomy


2. Unicompartmental knee replacement
3. High tibial osteotomy (HTO), lateral closing wedge
4. HTO, medial opening wedge with decreased tibial slope

PREFERRED RESPONSE: 4

DISCUSSION
This patient had a previous anterior cruciate ligament (ACL) and posterolateral complex injury. With
chronic instability and osteoarthritis, the best option is HTO with a decrease in the tibial slope to reduce
anterior laxity. Distal femoral osteotomy is better suited to address valgus malalignment. The lateral
closing-wedge osteotomy would not allow for adequate correction of the tibial slope. Unicompartmental
knee replacement is not indicated when there is ligament instability. If the patient continues to experience
instability following correction of the varus malalignment, reconstruction of the ACL and posterolateral
corner would be appropriate at that time.

RECOMMENDED READINGS
Rossi R, Bonasia DE, Amendola A. The role of high tibial osteotomy in the varus knee. J Am Acad Orthop
Surg. 2011 Oct;19(10):590-9. Review. PubMed PMID: 21980024.
Savarese E, Bisicchia S, Romeo R, Amendola A. Role of high tibial osteotomy in chronic injuries of
posterior cruciate ligament and posterolateral corner. J Orthop Traumatol. 2011 Mar;12(1):1-17. Epub
2010 Nov 24. Review. PubMed PMID: 21107635.

2013 American Academy of Orthopaedic Surgeons 2013 Sports Medicine Self-Assessment Examination
2013 Sports Medicine Self-Assessment Examination Answer Book 63

RESPONSES FOR QUESTIONS 69 THROUGH 71


1. Bone-patella tendon-bone autograft (10 mm)
2. Soft-tissue allograft (8 mm)
3. Quadruple hamstring autograft (10 mm)

For each of the following, please select the preferred response from the choices above.

Question 69
Shortest biologic incorporation after surgical reconstruction

PREFERRED RESPONSE: 1

Question 70
Highest ultimate tensile load (UTL)

PREFERRED RESPONSE: 3

Question 71
Highest risk for disease transmission

PREFERRED RESPONSE: 2

DISCUSSION FOR QUESTIONS 69 THROUGH 71


Anterior cruciate ligament (ACL) graft incorporation into bone follows a multiphase pattern. The first
phase is an inflammatory response with initial donor cell degeneration and provision of a scaffold for
host cell migration (occurs in up to 20 days). The second phase involves revascularization and host cell
fibroblast migration (20 days to 6 months). The final phase involves completion of graft healing and
remodeling into a more organized pattern of collagen structure. Bone-to-bone healing has been found
to have the shortest duration at approximately 6 weeks. Multiple studies have evaluated the UTL of
the intact ACL and various tissues used for ACL reconstruction. The quadruple hamstring autograft of
equivalent diameter has the highest UTL of the examples given at approximately 4000 N. In comparison,
the native ACL and the bone-tendon-bone autograft have a UTL of approximately 2100 N and 3000
N, respectively. Allograft carries the highest risk for disease transmission among the examples in this
question. These risks are low and largely eliminated with the screening guidelines developed and updated
by the American Association of Tissue Banks.

2013 American Academy of Orthopaedic Surgeons 2013 Sports Medicine Self-Assessment Examination
64 American Academy of Orthopaedic Surgeons

RECOMMENDED READINGS FOR QUESTIONS 69 THROUGH 71


West RV, Harner CD. Graft selection in anterior cruciate ligament reconstruction. J Am Acad Orthop Surg.
2005 May-Jun;13(3):197-207. Review. PubMed PMID: 15938608.
Noyes FR, Butler DL, Grood ES, Zernicke RF, Hefzy MS. Biomechanical analysis of human ligament
grafts used in knee-ligament repairs and reconstructions. J Bone Joint Surg Am. 1984 Mar;66(3):344-52.
PubMed PMID: 6699049.
Woo SL, Hollis JM, Adams DJ, Lyon RM, Takai S. Tensile properties of the human femur-anterior
cruciate ligament-tibia complex. The effects of specimen age and orientation. Am J Sports Med. 1991
May-Jun;19(3):217-25. PubMed PMID: 1867330.
Shelton WR, Treacy SH, Dukes AD, Bomboy AL. Use of allografts in knee reconstruction: I. Basic
science aspects and current status. J Am Acad Orthop Surg. 1998 May-Jun;6(3):165-8. Review. PubMed
PMID: 9682078.
Falconiero RP, DiStefano VJ, Cook TM. Revascularization and ligamentization of autogenous anterior
cruciate ligament grafts in humans. Arthroscopy. 1998 Mar;14(2):197-205. PubMed PMID: 9531133.

END OF SERIES

Figure 72

Question 72
Figure 72 is the MRI scan of a 61-year-old man who had left shoulder pain with a massive rotator cuff
tear. Active forward elevation was 120 degrees. Arthroscopic examination revealed that the rotator cuff
tear was irreparable. The articular surfaces of the glenohumeral joint have a normal appearance without
significant degenerative changes. What is the most appropriate treatment option?

1. Biceps tenotomy
2. Loose body removal
3. Latissimus dorsi transfer
4. Reverse total shoulder arthroplasty

PREFERRED RESPONSE: 1

2013 American Academy of Orthopaedic Surgeons 2013 Sports Medicine Self-Assessment Examination
2013 Sports Medicine Self-Assessment Examination Answer Book 65

DISCUSSION
The MRI scan shows medial subluxation of the biceps tendon. Biceps tenotomy has been an effective
treatment option for patients with large to massive rotator cuff tears when the tear is irreparable and pain
is the main symptom. The MRI scan does not show a loose body. Patients with severe external rotation
deficit and a deficient teres minor may experience a better functional result with latissimus dorsi transfer.
Reverse total shoulder arthroplasty is an option in patients with cuff tear arthropathy and pseudoparalysis.

RECOMMENDED READINGS
Getz CL, Buzzell JE, Krishnan SG. Shoulder instability and rotator cuff tears. In: Flynn JM, ed.
Orthopaedic Knowledge Update 10. Rosemont, IL: American Academy of Orthopaedic Surgeons;
2011;299-314.
Boileau P, Baqu F, Valerio L, Ahrens P, Chuinard C, Trojani C. Isolated arthroscopic biceps tenotomy or
tenodesis improves symptoms in patients with massive irreparable rotator cuff tears. J Bone Joint Surg
Am. 2007 Apr;89(4):747-57. PubMed PMID: 17403796.

CLINICAL SITUATION FOR QUESTIONS 73 THROUGH 75


A 17-year-old female swimmer has a 3-month history of increasing right shoulder pain. She reports
feeling as if her shoulder is "popping" out of place and pain with overhead activities. She also reports
nonspecific sensory changes in her arm on rare occasions. She has had to stop all sports activities because
her symptoms have progressed. On examination, sulcus and apprehension sign findings are positive. She
also has a positive Neer impingement sign. The Jobe empty can test and speed test results are negative.

Question 73
What structure is most likely associated with the etiology of this patients symptoms?

1. Rotator cuff
2. Biceps tendon
3. Capsule
4. Brachial plexus

PREFERRED RESPONSE: 3

Question 74
What is the most appropriate initial treatment for her condition?

1. Complete rest and a slow return to sports


2. Physical therapy
3. Corticosteroid injection
4. Arthroscopic surgery

PREFERRED RESPONSE: 2

2013 American Academy of Orthopaedic Surgeons 2013 Sports Medicine Self-Assessment Examination
66 American Academy of Orthopaedic Surgeons

Question 75
After 1 year of following recommended treatment, this patient continues to experience her symptoms and
has had to cease all sports activity. An MRI scan reveals no evidence of definitive labral or rotator cuff
pathology. At this stage, what is the most appropriate treatment option?

1. Arthroscopic rotator cuff repair


2. Arthroscopic biceps tenodesis
3. Arthroscopic capsular plication
4. Arthroscopic superior labral anterior-posterior repair

PREFERRED RESPONSE: 3

DISCUSSION FOR QUESTIONS 73 THROUGH 75


This patient has a history most consistent with multidirectional instability. A lax capsule causes
subluxation of the shoulder and strain on the rotator cuff and may result in pain and instability. The
capsule is most closely associated with the cause of her problem. Initial treatment for multidirectional
instability is physical therapy focusing on restoring balance to the shoulder with rotator cuff and scapular
stabilization exercises. Nonsurgical therapy should be protracted and is the mainstay of treatment in
this scenario. This patient has exhausted all nonsurgical measures and is now a candidate for surgical
reconstruction. Capsular plication will best address the lax capsule and provide the best option for
reducing her symptoms. The rotator cuff and biceps tendon may be secondarily strained but are not
the primary sources of the problem. The brachial plexus does not address the etiology, but rather the
symptoms that may occur as a result of instability of the shoulder joint. Complete rest will not alleviate
the patient's underlying condition because the shoulder girdle may still be weak and symptoms likely will
return. A corticosteroid injection and arthroscopic surgery are too invasive as initial treatment for this
condition. Arthroscopic rotator cuff repair, a biceps tenodesis, and superior labral anterior-posterior repair
are unlikely to result in symptomatic improvement for this patient and are not associated with pathologic
findings in the setting of multidirectional instability.

RECOMMENDED READINGS FOR QUESTIONS 73 THROUGH 75


Tjoumakaris FP, Bradley JP. The rationale for an arthroscopic approach to shoulder stabilization.
Arthroscopy. 2011 Oct;27(10):1422-33. Epub 2011 Aug 26. Review. PubMed PMID: 21872422.
Gaskill TR, Taylor DC, Millett PJ. Management of multidirectional instability of the shoulder. J Am Acad
Orthop Surg. 2011 Dec;19(12):758-67. Review. PubMed PMID: 22134208.

END OF SERIES

2013 American Academy of Orthopaedic Surgeons 2013 Sports Medicine Self-Assessment Examination
2013 Sports Medicine Self-Assessment Examination Answer Book 67

Figure 76a Figure 76b Figure 76c Figure 76d

Question 76
What do the T2-weighted, fat-saturated MRI scans shown in Figures 76a through 76d reveal?

1. Posterior cruciate ligament (PCL) tear, isolated


2. PCL tear and medial meniscus tear
3. Anterior cruciate ligament (ACL) tear, isolated
4. ACL tear and medial meniscus tear

PREFERRED RESPONSE: 4

DISCUSSION
The MRI scans show that edema is noted on the femoral insertion of the ACL consistent with a high-grade
or complete ACL tear. The ACL is not visualized on the sagittal view, although the torn meniscus can be
seen in the notch. On the coronal image, there is an empty lateral wall sign indicating proximal disruption
of the ACL. The medial meniscus images show a disruption of normal meniscus morphology consistent
with a bucket handle medial meniscus tear. Note the appearance on the sagittal MRI scan of what appears
to be a second soft-tissue density in line with the PCL. This double PCL sign is highly indicative of a
displaced medial meniscus tear rather than a displaced lateral meniscus tear.

RECOMMENDED READINGS
Weiss KL, Morehouse HT, Levy IM. Sagittal MR images of the knee: a low-signal band parallel to
the posterior cruciate ligament caused by a displaced bucket-handle tear. AJR Am J Roentgenol. 1991
Jan;156(1):117-9. PubMed PMID: 1898543.
Munk B, Madsen F, Lundorf E, Staunstrup H, Schmidt SA, Bolvig L, Hellfritzsch MB, Jensen J. Clinical
magnetic resonance imaging and arthroscopic findings in knees: a comparative prospective study of
meniscus anterior cruciate ligament and cartilage lesions. Arthroscopy. 1998 Mar;14(2):171-5. PubMed
PMID: 9531128.

2013 American Academy of Orthopaedic Surgeons 2013 Sports Medicine Self-Assessment Examination
68 American Academy of Orthopaedic Surgeons

Question 77
A 28-year-old woman underwent a closing-wedge high tibial osteotomy (HTO) for medial compartment
overload after medial meniscectomy. Postsurgically, she reported improvement in her medial pain and
resumed normal activities. About 9 months after her surgery, however, she reports burning pain in the
front of her knee with running. Her examination reveals no joint line tenderness, mild pain with patellar
compression, and limited patellar glides. What is the most likely cause of her symptoms?

1. Patella infera (baja)


2. Patella alta
3. Recurrence of medial joint overload
4. Nonunion of the osteotomy

PREFERRED RESPONSE: 1

DISCUSSION
After HTO, particularly in patients who have been immobilized after a closing-wedge osteotomy, patella
baja is a common finding. This can precipitate anterior knee pain or patellofemoral pain syndrome.
Recurrence of medial joint overload is incorrect because the patient has no medial joint complaints.
Nonunion is less likely with a closing-wedge osteotomy and likely will not result in anterior knee pain.

RECOMMENDED READINGS
Scuderi GR, Windsor RE, Insall JN. Observations on patellar height after proximal tibial osteotomy. J
Bone Joint Surg Am. 1989 Feb;71(2):245-8. PubMed PMID: 2918009.
Aglietti P, Buzzi R, Vena LM, Baldini A, Mondaini A. High tibial valgus osteotomy for medial
gonarthrosis: a 10- to 21-year study. J Knee Surg. 2003 Jan;16(1):21-6. PubMed PMID: 12568262.

CLINICAL SITUATION FOR QUESTIONS 78 THROUGH 80


A 14-year-old pitcher who plays in an extended-season baseball program has vague pain in his dominant
shoulder. The pain occurs with throwing and it has been worsening during the past 2 months. External
rotation in abduction is 140 degrees in the dominant arm and 130 degrees in the nondominant arm.
Internal rotation in abduction is 30 degrees in the dominant arm and 40 degrees in the nondominant arm.
The sulcus sign is negative. Radiographs show subtle widening of the proximal humeral physis.

Question 78
What is the most likely diagnosis?

1. Little leaguers shoulder


2. Glenohumeral internal rotation deficit
3. Rotator cuff tendonitis
4. Multidirectional instability

PREFERRED RESPONSE: 1

2013 American Academy of Orthopaedic Surgeons 2013 Sports Medicine Self-Assessment Examination
2013 Sports Medicine Self-Assessment Examination Answer Book 69

Question 79
Initial treatment should consist of

1. arthroscopic capsular plication.


2. rest until symptoms resolve.
3. manipulation under anesthesia.
4. closed reduction and percutaneous pinning.

PREFERRED RESPONSE: 2

Question 80
What is the etiology of this patients problem?

1. Ischemia
2. Overuse condition
3. Pathologic laxity
4. Increased humeral retroversion

PREFERRED RESPONSE: 2

DISCUSSION FOR QUESTIONS 78 THROUGH 80


Little leaguers shoulder is an overuse condition of the proximal humeral physis. Patients report diffuse
pain that is worse with throwing. Factors that contribute to the condition include excessive throwing,
improper throwing mechanics, and muscle-tendon imbalance. Radiographs usually show widening of
the proximal humeral physis, and, in more severe cases, metaphyseal demineralization or fragmentation.
Surgical fixation is not required for healing. Treatment involves rest until symptoms resolve, followed
by initiation of an interval throwing program. Pitching coaches should evaluate throwing mechanics and
maintain pitch counts. The dominant shoulders of throwing athletes undergo adaptive changes, resulting
in increased external rotation and decreased internal rotation. These changes occur secondary to soft-
tissue and bony adaptations, including increased humeral retroversion. Ischemia has been implicated as
a potential cause of osteochondritis dissecans lesions. Rotator cuff tendonitis presents with anterolateral
shoulder pain that is worse with activity. Pain is reproduced with resisted supraspinatus testing, and
radiograph findings are typically normal. Adolescent shoulder injuries are often caused by subtle,
atraumatic instability most often sustained in sports with overhead movements, including baseball. These
symptoms are more likely to occur in athletes with evidence of increased soft-tissue laxity.

RECOMMENDED READINGS FOR QUESTIONS 78 THROUGH 80


Shea K, Ganley TJ. Injuries and conditions of the pediatric and adolescent athlete. In: Flynn JM, ed.
Orthopaedic Knowledge Update 10. Rosemont, IL: American Academy of Orthopaedic Surgeons;
2011:789-796.
Chen FS, Diaz VA, Loebenberg M, Rosen JE. Shoulder and elbow injuries in the skeletally immature
athlete. J Am Acad Orthop Surg. 2005 May-Jun;13(3):172-85. Review. PubMed PMID: 15938606.

END OF SERIES

2013 American Academy of Orthopaedic Surgeons 2013 Sports Medicine Self-Assessment Examination
70 American Academy of Orthopaedic Surgeons

Question 81
The sports-related concussion treatment guidelines regarding same-day return to play developed during
the 2008 3rd International Conference on Concussion in Sport in Zurich, Switzerland, recommend that

1. athletes with resolution of symptoms after an injury may return to play.


2. athletes with resolution of symptoms after an injury may not return to play.
3. a CT scan should be used for early diagnosis of concussion.
4. a noncontact athlete with symptoms at rest or with exertion should continue to play.

PREFERRED RESPONSE: 2

DISCUSSION
The Consensus Statement on Concussion in Sport was developed at the 3rd International Conference on
Concussion in Sport in Zurich in November 2008. Although the authors acknowledged that the science of
concussion was evolving, consensus existed regarding the principal messages conveyed at this conference.
Specifically, the panel agreed that athletes with symptoms should not be returned to play on the same day
and that subsequent serial monitoring and re-examination should be performed. Even if symptoms resolve
during a game, an injured athlete should not return to play on the same day. These guidelines apply to
both minimal-contact and noncontact athletes. Panel members believed that evaluation and management
recommendations in these guidelines could apply to children age 10 years and older. For children younger
than age 10, reported concussion symptoms differ from those experienced by adults, and age-appropriate
symptom checklists are required. Young elite athletes also should be treated with a more conservative
approach. The use of CT scan in early diagnosis of concussion was considered to contribute little to
concussion evaluation. However, if, based on serial examination, suspicion of an intracerebral structural
lesion is suspected, a CT scan should be employed. A multidisciplinary approach including a sideline
physician, trainers, parents, and consulting physicians was considered helpful and was recommended.

RECOMMENDED READINGS
Putukian M. The acute symptoms of sport-related concussion: diagnosis and on-field management. Clin
Sports Med. 2011 Jan;30(1):49-61, viii. Review. PubMed PMID: 21074081.
McCrory P, Meeuwisse W, Johnston K, Dvorak J, Aubry M, Molloy M, Cantu R. Consensus statement on
concussion in sport - the Third International Conference on Concussion in Sport held in Zurich, November
2008. Phys Sportsmed. 2009 Jun;37(2):141-59. Review. PubMed PMID: 20048521.
Prabhu SP. The role of neuroimaging in sport-related concussion. Clin Sports Med. 2011 Jan;30(1):103-
14, ix. Review. PubMed PMID: 21074085.

2013 American Academy of Orthopaedic Surgeons 2013 Sports Medicine Self-Assessment Examination
2013 Sports Medicine Self-Assessment Examination Answer Book 71

Figure 82

Question 82
Figure 82 is the MRI scan of a 15-year-old boy who has had knee pain with running for 5 months.
Radiographs show an osteochondritis dissecans (OCD) lesion of the medial femoral condyle. What is the
most appropriate treatment?

1. Arthroscopic or open reduction and internal fixation


2. Arthroscopic loose body removal
3. Activity restriction for up to 9 months
4. Subchondral drilling

PREFERRED RESPONSE: 1

DISCUSSION
OCD is an acquired lesion of the subchondral bone. Patients with OCD initially report nonspecific pain
and variable amounts of swelling. Initial radiographs help identify the lesion and establish the status
of the physes. An MRI scan is useful for assessing the potential for the lesion to heal with nonsurgical
treatment. Nonsurgical treatment is appropriate for small, stable lesions in patients with open physes and
focuses on activity restriction for 3 to 9 months. Surgical treatment is necessary for unstable or detached
lesions. Stable lesions with intact articular cartilage can be treated with subchondral drilling to stimulate
vascular ingrowth, with radiographic healing at an average of 4.4 months. Fixation is indicated for
unstable or hinged lesions, and stabilization of the fragment can be achieved using a variety of implants
through an arthroscopic or open approach. The fragment should be salvaged and the normal articular
surface restored whenever possible.

RECOMMENDED READINGS
Shea K, Ganley TJ. Injuries and conditions of the pediatric and adolescent athlete. In: Flynn JM, ed.
Orthopaedic Knowledge Update 10. Rosemont, IL: American Academy of Orthopaedic Surgeons;
2011:783-796.
Magnussen RA, Carey JL, Spindler KP. Does operative fixation of an osteochondritis dissecans loose body
result in healing and long-term maintenance of knee function? Am J Sports Med. 2009 Apr;37(4):754-9.
Epub 2009 Feb 9. PubMed PMID: 19204369.

2013 American Academy of Orthopaedic Surgeons 2013 Sports Medicine Self-Assessment Examination
72 American Academy of Orthopaedic Surgeons

CLINICAL SITUATION FOR QUESTIONS 83 AND 84


A 6-ft, 6-in white man has undergone 3 previous open right-knee surgeries. He recently learned he had an
intra-articular loose body and underwent an arthrotomy with loose body removal. At surgery, 2 previous
skin incisions were noted: midline and medial parapatellar. The orthopaedic surgeon uses the midline
incision for the arthrotomy and loose body removal. At the same time, the surgeon performs a lateral
release. The patient required 110 minutes of tourniquet time at a pressure of 325 mm. Postsurgically,
a cold compression device was applied over an island dressing and the patient continuously wore a
thromboembolic-deterrent stocking for 3 days. Three days after surgery the patient complained of burning
over the anterior aspect of his knee. He is afebrile and denies chills. Upon inspection, the wound has
blanching over the anterior knee and several clear blisters are present. There is no erythema or purulent
drainage from the wound. The knee has no effusion and good peripheral pulses are palpated.

Question 83
The appearance of the patients wound is most likely the result of

1. a postoperative infection.
2. a hypersensitivity reaction to the dressing.
3. frostbite injury secondary to continuous cold exposure.
4. frostbite injury with superimposed secondary infection.

PREFERRED RESPONSE: 3

Question 84
What is the most appropriate initial treatment?

1. Run a wound culture and return to the operating room for arthrotomy, irrigation, and
debridement.
2. Return to the operating room, blister debridement, split-thickness skin grafting, and
prophylactic intravenous (IV) penicillin G (PCN G) every 6 hours
3. Return to the operating room, blister debridement, apply topical aloe, and prophylactic IV
PCN G every 6 hours
4. Bedside blister debridement, apply topical aloe, and prophylactic IV PCN G every 6 hours

PREFERRED RESPONSE: 4

2013 American Academy of Orthopaedic Surgeons 2013 Sports Medicine Self-Assessment Examination
2013 Sports Medicine Self-Assessment Examination Answer Book 73

DISCUSSION FOR QUESTIONS 83 AND 84


Based upon the history and description of the wound, this patient has sustained a frostbite injury to the
anterior skin without evidence of infection. Frostbite, a thermal injury to local tissues, can be classified
into first-, second-, third-, and fourth-degree injuries. First-degree injuries are characterized by a central
whitish area surrounded by erythema. Second- and third-degree injuries are characterized by blisters that
appear within the first 24 hours. Second-degree blisters are clear or cloudy, while third-degree blisters
are hemorrhagic. Fourth-degree injuries are characterized by tissue necrosis. Treatment of a frostbite
injury should begin as soon as it is identified, even if severity has not been determined. Treatment
includes protection against mechanical irritation and keeping the injured area away from heat. The limb
should be elevated and clear blisters debrided; dark blisters should be drained but not debrided. Topical
treatment includes aloe application every 6 hours (silver sulfadiazine for open wounds). Nonsteroidal
anti-inflammatory drugs can be given along with PCN G (500,000 U) or clindamycin (600 mg) IV every 6
hours for 48 to 72 hours as prophylaxis against infection.

Multiple skin incisions on the front of the knee place the skin at healing risk after surgery. Skin
oxygenation is further compromised by a lateral release, which disrupts the superior lateral geniculate
artery in the peripatellar vascular anastomosis. When addressing compromised perfusion to the skin, the
orthopaedic surgeon may consider not performing a lateral release, using the medial parapatellar incision,
and/or not using a tourniquet during surgery. In this patient, the use of cold therapy further compromised
perfusion to the skin. This patients wound does not reflect infection, but rather frostbite from the
compromised skin perfusion and the application of cold therapy.

The physiologic responses to the effects of freezing temperatures on limbs have been categorized into 4
phases. Phase I, cooling and freezing, is characterized by vasoconstriction and vasospasm. The freezing
results in mechanical destruction of cell membranes in endothelial cells in small capillaries. Phase II,
rewarming, initiates as heat absorbed by the limb begins an exothermic reaction as extracellular and
intracellular crystals melt. Intracellular swelling occurs and small capillaries become highly permeable,
resulting in extravasation of fluid and causing edema and blisters. Phase III is characterized by
progressive tissue injury resulting from inflammation, vascular stasis, and thrombosis leading to ischemia.
Phase IV is recovery that can progress along 3 potential pathways: complete healing, healing with later
sequelae, or early tissue necrosis leading to gangrene.

RECOMMENDED READINGS FOR QUESTIONS 83 AND 84


Golant A, Nord RM, Paksima N, Posner MA. Cold exposure injuries to the extremities. J Am Acad Orthop
Surg. 2008 Dec;16(12):704-15. Review. PubMed PMID: 19056919.
Johnson DP. Midline or parapatellar incision for knee arthroplasty. A comparative study of wound
viability. J Bone Joint Surg Br. 1988 Aug;70(4):656-8. PubMed PMID: 3403619.
Cauchy E, Marsigny B, Allamel G, Verhellen R, Chetaille E. The value of technetium 99 scintigraphy
in the prognosis of amputation in severe frostbite injuries of the extremities: A retrospective study of 92
severe frostbite injuries. J Hand Surg Am. 2000 Sep;25(5):969-78. PubMed PMID: 11040315.

END OF SERIES

2013 American Academy of Orthopaedic Surgeons 2013 Sports Medicine Self-Assessment Examination
74 American Academy of Orthopaedic Surgeons

Figure 85

CLINICAL SITUATION FOR QUESTIONS 85 THROUGH 87


Figure 85 is the radiograph of a 13-year-old right-handed baseball player who has experienced 2 months
of right-dominant shoulder pain. As a pitcher, he says he cannot throw hard without pain, and he
develops a dull ache if he throws more than 15 pitches.

Question 85
Where is this injury located?

1. Hypertrophic zone of the physis


2. Posterior superior humeral head
3. Surgical neck of the humerus
4. Resting cartilage zone of the physis

PREFERRED RESPONSE: 1

Question 86
What is the diagnosis?

1. Osteonecrosis
2. Proximal humerus epiphysiolysis
3. Hill-Sachs impaction fracture from subluxation
4. Salter-Harris type II fracture of the proximal humerus

PREFERRED RESPONSE: 2

2013 American Academy of Orthopaedic Surgeons 2013 Sports Medicine Self-Assessment Examination
2013 Sports Medicine Self-Assessment Examination Answer Book 75

Question 87
What is the recommended treatment for this injury?

1. Bankart repair
2. Hanging arm cast
3. Rest and no pitching for 3 months
4. Resume a throwing program but avoid curveballs and sliders

PREFERRED RESPONSE: 3

DISCUSSION FOR QUESTIONS 85 THROUGH 87


The hypertrophic zone of the growth plate has been implicated as the weak link in the physis in acute
injuries. Epiphysiolysis of the proximal humerus in throwing athletes occurs as the result of tension and
shear on the physis. More than 90% of affected patients who are treated with rest for an average of 3
months become asymptomatic. Prevention is the best option. Set limitations of the number of pitches and
types of pitches depending on the age of the player. Also recommend use of proper pitching mechanics.

RECOMMENDED READINGS FOR QUESTIONS 85 THROUGH 87


Carson WG Jr, Gasser SI. Little Leaguers shoulder. A report of 23 cases. Am J Sports Med. 1998 Jul-
Aug;26(4):575-80. PubMed PMID: 9689382.
Chen FS, Diaz VA, Loebenberg M, Rosen JE. Shoulder and elbow injuries in the skeletally immature
athlete. J Am Acad Orthop Surg. 2005 May-Jun;13(3):172-85. Review. PubMed PMID: 15938606.
Lyman S, Fleisig GS, Andrews JR, Osinski ED. Effect of pitch type, pitch count, and pitching
mechanics on risk of elbow and shoulder pain in youth baseball pitchers. Am J Sports Med. 2002 Jul-
Aug;30(4):463-8. PubMed PMID: 12130397.

END OF SERIES

Question 88
A 45-year-old postmenopausal smoker with a body mass index (BMI) of 22 has had severe knee pain
for the past year. The pain has been progressing and the patient is now only able to perform activities of
daily living. Knee radiographs reveal medial compartment osteoarthritis without any involvement of the
patellofemoral joint or the lateral compartment. What is the contraindication for a high tibial osteotomy
(HTO) in this patient?

1. Smoking status
2. Postmenopausal status
3. BMI
4. Radiographic findings

PREFERRED RESPONSE: 1

2013 American Academy of Orthopaedic Surgeons 2013 Sports Medicine Self-Assessment Examination
76 American Academy of Orthopaedic Surgeons

DISCUSSION
The principal contraindications to valgus-producing HTO include (1) lateral compartment degenerative
joint disease, (2) loss of a significant portion of the lateral meniscus, (3) symptomatic patellofemoral
degenerative joint disease, (4) nonconcordant pain (ie, patellofemoral pain with medial compartment
osteoarthritis), (5) smoking, (6) patient unwillingness to accept the anticipated cosmetic appearance of the
desired amount of angular correction, and (7) inflammatory arthritis.

RECOMMENDED READINGS
Amendola A, Bonasia DE. Results of high tibial osteotomy: review of the literature. Int Orthop. 2010
Feb;34(2):155-60. Epub 2009 Oct 17. Review. PubMed PMID: 19838706.
Savarese E, Bisicchia S, Romeo R, Amendola A. Role of high tibial osteotomy in chronic injuries of
posterior cruciate ligament and posterolateral corner. J Orthop Traumatol. 2011 Mar;12(1):1-17. Epub
2010 Nov 24. Review. PubMed PMID: 21107635.

Figure 89a Figure 89b Figure 89c

Figure 89d Figure 89e

CLINICAL SITUATION FOR QUESTIONS 89 THROUGH 95


An 18-year-old high school basketball player sustains a noncontact injury to his right knee while landing
after a rebound. He reports hearing a pop and has immediate pain and the inability to bear weight on
that leg. During the next few hours he develops an effusion, which is aspirated by the team physician and
is found to be a lipohemarthrosis. After aspiration, the athlete is not able to perform a straight-leg raise
and a detailed examination is not possible because of guarding and pain.

2013 American Academy of Orthopaedic Surgeons 2013 Sports Medicine Self-Assessment Examination
2013 Sports Medicine Self-Assessment Examination Answer Book 77

Question 89
What is the next treatment step?

1. Place the patient in a telescoping hinged knee brace that is unlocked and allow weight bearing
as tolerated with crutches for support.
2. Place the patient in a telescoping hinged knee brace locked in extension with toe-touch weight
bearing.
3. Place the patient in a neoprene sleeve and allow weight bearing as tolerated.
4. Place the patient in a neoprene sleeve with toe-touch weight bearing.

PREFERRED RESPONSE: 2

Question 90
Radiographs are shown in Figures 89a through 89c. What is the most likely diagnosis?

1. Medial tibial plateau fracture


2. Chronic medial collateral ligament injury
3. Anterior cruciate ligament (ACL) rupture
4. Acute lateral collateral ligament rupture

PREFERRED RESPONSE: 3

Question 91
Figures 89d and 89e are this patients T2-weighted sagittal MRI scans. What is most commonly
associated with these MRI findings?

1. Medial meniscus tear


2. Lateral meniscus tear
3. Posterolateral corner (PLC) injury
4. Posterior cruciate ligament (PCL) rupture

PREFERRED RESPONSE: 2

2013 American Academy of Orthopaedic Surgeons 2013 Sports Medicine Self-Assessment Examination
78 American Academy of Orthopaedic Surgeons

Question 92
This athletes MRI scan reveals an acute ACL tear. He has accepted a scholarship to play basketball at an
NCAA Division I school. What is the role of the team physician in reporting this injury to the scholarship
school?

1. Encourage the athlete to report his injury and treatment to the scholarship schools coaching
staff.
2. Encourage the athlete not to discuss the injury with his scholarship school because this might
endanger his scholarship.
3. Directly contact the scholarship schools medical staff and report the injury and treatment
rendered.
4. Discuss the injury with the athletes parents and allow them to make a decision about how to
proceed.

PREFERRED RESPONSE: 1

Question 93
The athlete is taken to the operating room for arthroscopic evaluation and treatment. While the patient is
under anesthesia, the knee is found to have full motion with a grade 2B Lachman examination, a positive
pivot shift, 1+ posterior drawer, and equivalent external rotation of the tibia in 30 degrees and 90 degrees
of flexion. The examination is consistent with what injury?

1. Isolated incomplete ACL rupture


2. Complete ACL rupture
3. Complete ACL rupture with posterolateral corner injury
4. Isolated posterolateral corner injury

PREFERRED RESPONSE: 2

Question 94
When compared to double-bundle ACL reconstruction, what is the disadvantage of traditional trans-tibial
single-bundle ACL reconstruction?

1. Less anterior-posterior stability


2. Less rotational stability
3. Higher cost
4. Longer surgical time

PREFERRED RESPONSE: 2

2013 American Academy of Orthopaedic Surgeons 2013 Sports Medicine Self-Assessment Examination
2013 Sports Medicine Self-Assessment Examination Answer Book 79

Question 95
Postsurgically, the patient recovers well and is fully rehabilitated. He demonstrates full motion with no
instability or pain and is cleared to return to play 12 months after the surgery. He asks for your advice
regarding use of a functional brace for playing basketball following his reconstruction. What is the most
appropriate recommendation?

1. The athlete must wear a functional brace for all athletic activities for 2 years following
reconstruction.
2. The athlete may wear a functional brace for athletic activities; however, no evidence exists to
show the brace decreases the rate of ACL retear.
3. The athlete must wear a functional brace for 2 years following reconstruction for basketball
only; other athletic activities such as running and tennis are allowed without the brace.
4. The athlete must wear a custom-fit functional brace for 2 years following reconstruction
because off-the-shelf braces produce inferior results.

PREFERRED RESPONSE: 2

DISCUSSION FOR QUESTIONS 89 THROUGH 95


The athlete most likely suffered an acute ACL rupture however the presence of a lipohemarthrosis is
concerning for the possibility of an intraarticular fracture. Because of this, the patient should not be
allowed full weight bearing until a fracture is ruled out with radiographs. Given the athletes inability
to perform a straight leg raise, the extensor mechanism is not functioning and a telescoping knee brace
locked in extension should be utilized. A neoprene knee sleeve does not have a role in the treatment of
this acute injury.

The radiographs reveal a lateral avulsion fracture off of the proximal tibial epiphysis which is known as a
Segond fracture. It is indicative of an ACL injury and the fracture fragment seen is the consequence of the
lateral capsule injury sustained during the pivot-shift mechanism. There is no radiographic evidence of a
medial tibial plateau fracture. A radiographic sign of a chronic MCL injury is known as a Pellegrini-Stieda
lesion and this is seen as calcification of the femoral origin of the MCL. A radiographic sign of an acute
LCL rupture would be an avulsion fracture of the tip of the fibula.

The MRI shows kissing contusions of the posterolateral tibial plateau and the midpoint of the lateral
femoral condyle. These kissing lesions are seen as a result of a pivot shift mechanism of injury and are
diagnostic for an ACL rupture. The most common associated injury in an acute ACL rupture is a lateral
meniscus tear. Medial meniscus tears are more common in chronic ACL injuries. PCL rupture and PLC
injury are all associated injuries seen in acute ACL rupture; however, these are much less common than
meniscal tears.

The team physician has a role in encouraging, but not demanding, the athlete to report the injury and
treatment to the scholarship schools coaching staff. What the athlete decides to do is his decision; the
physician would be violating the athletes HIPAA rights as well as their confidence by reporting it directly
to the scholarship school. Clearly the physician should not discourage the athlete from reporting the
injury. The athlete is 18 years old and, as such, the physician would need the athletes permission to
discuss any medical issues with the family in keeping with HIPAA.

2013 American Academy of Orthopaedic Surgeons 2013 Sports Medicine Self-Assessment Examination
80 American Academy of Orthopaedic Surgeons

The athletes exam demonstrates incompetence of both bundles of the ACL as demonstrated by the loss
of stability with anterior translation of the tibia (Lachman test) as well as with rotation (pivot shift). The
external rotation stress with the knee in 30 degrees of flexion tests the competence of the posterolateral
corner while rotation at 90 degrees of flexion tests the PCL. Since the athletes knee is stable to posterior
drawer testing demonstrating an intact PCL and the external rotation at 30 degrees is equivalent to that at
90 degrees, the posterolateral corner in intact.

The ACL has two separate and distinct bundles, the AM and PL. Each bundle takes on tension at varying
degrees of knee flexion and therefore each bundle is thought to have a varying contribution to the stability
of the knee. The AM bundle takes on tension with the knee in flexion and the PL bundle is tight in
extension. Neither bundle is isometric during knee range of motion. Both bundles have contributions to
rotational stability of the knee throughout the range of motion.

The success of traditional trans-tibial single-bundle ACL reconstruction has recently been called into
question given the demonstration of persistent rotational instability following reconstruction. The
persistence of rotational instability in trans-tibial single bundle ACL reconstruction has been attributed
to the location of the graft in a vertically malpositioned femoral tunnel. The goal of double-bundle
ACL reconstruction is to more accurately reproduce the native ACL and provide grafts that contribute to
anteroposterior stability as well as rotational stability by placing the grafts in more anatomic locations
not central in the knee axis. There is an increased cost and surgical time associated with double-bundle
reconstruction.

The use of functional braces following ACL reconstruction is a surgeons preference because there is no
difference in retear rate with or without a brace. Some authors recommend brace use for one to two years
following ACL reconstruction for all athletic activities, but this is not supported by the literature. No
literature exists showing a higher rate of reinjury with a functional brace and off-the shelf and custom
braces have been found to be equivalent leading those who advocate for braces to recommend off-the-
shelf braces given their significantly lower cost.

2013 American Academy of Orthopaedic Surgeons 2013 Sports Medicine Self-Assessment Examination
2013 Sports Medicine Self-Assessment Examination Answer Book 81

RECOMMENDED READINGS FOR QUESTIONS 89 THROUGH 95


Ma CB, Rodeo SA. Meniscal injuries. In: Garrick JG, ed. Orthopaedic Knowledge Update: Sports
Medicine 3. Rosemont, IL: American Academy of Orthopaedic Surgeons; 2004:199-212.
Johnson TS, Cosgarea AJ. Posterior cruciate ligament injuries. In: Garrick JG, ed. Orthopaedic Knowledge
Update: Sports Medicine 3. Rosemont, IL: American Academy of Orthopaedic Surgeons; 2004:155-168.
Bellabarba C, Bush-Joseph CA, Bach BR Jr. Patterns of meniscal injury in the anterior cruciate-deficient
knee: a review of the literature. Am J Orthop (Belle Mead NJ). 1997 Jan;26(1):18-23. Review. PubMed
PMID: 9021030.
Gottsegen CJ, Eyer BA, White EA, Learch TJ, Forrester D. Avulsion fractures of the knee: imaging
findings and clinical significance. Radiographics. 2008 Oct;28(6):1755-70. Review. PubMed PMID:
18936034.
Goldman AB, Pavlov H, Rubenstein D. The Segond fracture of the proximal tibia: a small avulsion that
reflects major ligamentous damage. AJR Am J Roentgenol. 1988 Dec;151(6):1163-7. PubMed PMID:
3263770.
Magee J T, Almekinders L C, Taft T N. HIPAA and the team physician. Sports Medicine Update: March
April; 2003:4-7.
Hill D. A matter of privacy. Athletic Management. 2003;15(2):37-42.
Lane CG, Warren R, Pearle AD. The pivot shift. J Am Acad Orthop Surg. 2008 Dec;16(12):679-88.
Review. PubMed PMID: 19056917.
Zelle BA, Vidal AF, Brucker PU, Fu FH. Double-bundle reconstruction of the anterior cruciate ligament:
anatomic and biomechanical rationale. J Am Acad Orthop Surg. 2007 Feb;15(2):87-96. Review. PubMed
PMID: 17277255.
Sakane M, Fox RJ, Woo SL, Livesay GA, Li G, Fu FH. In situ forces in the anterior cruciate ligament and
its bundles in response to anterior tibial loads. J Orthop Res. 1997 Mar;15(2):285-93. PubMed PMID:
9167633.
Woo SL, Kanamori A, Zeminski J, Yagi M, Papageorgiou C, Fu FH. The effectiveness of reconstruction of
the anterior cruciate ligament with hamstrings and patellar tendon . A cadaveric study comparing anterior
tibial and rotational loads. J Bone Joint Surg Am. 2002 Jun;84-A(6):907-14. PubMed PMID: 12063323.
Wright RW, Fetzer GB. Bracing after ACL reconstruction: a systematic review. Clin Orthop Relat Res.
2007 Feb;455:162-8. Review. PubMed PMID: 17279043.
Birmingham TB, Bryant DM, Giffin JR, Litchfield RB, Kramer JF, Donner A, Fowler PJ. A randomized
controlled trial comparing the effectiveness of functional knee brace and neoprene sleeve use after anterior
cruciate ligament reconstruction. Am J Sports Med. 2008 Apr;36(4):648-55. Epub 2008 Jan 11. PubMed
PMID: 18192493.

END OF SERIES

2013 American Academy of Orthopaedic Surgeons 2013 Sports Medicine Self-Assessment Examination
82 American Academy of Orthopaedic Surgeons

Figure 96a Figure 96b

Question 96
Figures 96a and 96b are the MRI scans of a 57-year-old man who dislocated his left shoulder after a fall
while playing tennis. On examination he had full passive shoulder range of motion, but he was unable to
actively elevate his injured shoulder. Sensation was intact to light touch over the lateral shoulder. What is
the most likely etiology of his shoulder weakness?

1. Axillary nerve injury


2. Cervical radiculopathy involving the C6 nerve root
3. Massive rotator cuff tear with loss of the transverse force couple
4. Long head of the biceps tendon rupture with loss of superior stabilizing effect

PREFERRED RESPONSE: 3

DISCUSSION
This patient has a massive rotator cuff tear resulting in disruption of the transverse force couple between
the subscapularis anteriorly and the infraspinatus and teres minor posteriorly. These muscles provide
dynamic shoulder stability throughout active elevation, and loss of the force couple produces a pathologic
increase in translation of the humeral head and decreased active abduction. Active shoulder elevation
less than 90 degrees in the presence of full passive motion is termed pseudoparalysis. The most common
neurologic deficit after shoulder dislocation is isolated injury to the axillary nerve. This patients sensory
examination suggests that the axillary nerve is intact. Cervical radiculopathy is less common after
shoulder dislocation but has been reported. Conflicting evidence exists regarding the contribution of the
long head of the biceps tendon to glenohumeral stability. One study reported minimal electromyographic
activity in the biceps during 10 basic shoulder motions.

RECOMMENDED READINGS
Getz CL, Buzzell JE, Krishnan SG. Shoulder instability and rotator cuff tears. In: Flynn JM, ed.
Orthopaedic Knowledge Update 10. Rosemont, IL: American Academy of Orthopaedic Surgeons;
2011:299-314.
Robinson CM, Shur N, Sharpe T, Ray A, Murray IR. Injuries associated with traumatic anterior
glenohumeral dislocations. J Bone Joint Surg Am. 2012 Jan 4;94(1):18-26. PubMed PMID: 22218378.

2013 American Academy of Orthopaedic Surgeons 2013 Sports Medicine Self-Assessment Examination
2013 Sports Medicine Self-Assessment Examination Answer Book 83

CLINICAL SITUATION FOR QUESTIONS 97 AND 98


A 14-year-old gymnast missed her dismount off of the uneven bars, hit the mat face first, and lost
consciousness for about 15 seconds. She was dazed and confused for several minutes. She does not
complain of pain, numbness, or weakness and she is moving all extremities without deficit.

Question 97
The athlete and coach want to go back to competition that day. How should they be advised?

1. Concussion precludes same-day return to play.


2. Order an urgent MRI scan; if findings are normal, she can return to competition.
3. Order neurocognitive testing; if findings are normal, she can return to competition.
4. If she is symptom-free after a 15-minute exertional test, she may return to competition.

PREFERRED RESPONSE: 1

Question 98
Thirty minutes later the gymnast is experiencing headache and difficulty concentrating. If her symptoms
persist 1 week later, the next treatment step should be

1. a functional MRI scan.


2. serial neurocognitive testing.
3. no return to play that season.
4. cognitive, cranial nerve, and balance testing after a period of moderate-to-intense exercise.

PREFERRED RESPONSE: 2

DISCUSSION FOR QUESTIONS 97 AND 98


The National Collegiate Athletic Associations (NCAA) 2011 revised health and safety guidelines
regarding concussion management (available at www.ncaa.org) recommend no return to play on the same
day of an injury. In particular, athletes sustaining a concussion should not return to play the same day as
their injury. Before resuming exercise, athletes must be asymptomatic or returned to baseline symptoms
at rest and have no symptoms with cognitive effort. They must be off of medications that could mask or
alter concussion symptoms. Neurocognitive testing can be a helpful tool in determining brain function
even after all symptoms of concussion have resolved. With a comparison baseline test, this evaluation, in
conjunction with a physicians examination, may reduce risk for second impact syndrome. The athletes
clinical neurological examination findings (cognitive, cranial nerve, and balance testing) must return to
baseline before resuming exercise. Research has shown that among youth athletes it may take longer for
tested functions to return to baseline (compared to the recovery rate in adult athletes). Brain MRI scan has
no role in evaluating athletes for return to play in this situation.

2013 American Academy of Orthopaedic Surgeons 2013 Sports Medicine Self-Assessment Examination
84 American Academy of Orthopaedic Surgeons

RECOMMENDED READINGS FOR QUESTIONS 97 AND 98


Herring SA, Cantu RC, Guskiewicz KM, Putukian M, Kibler WB, Bergfeld JA, Boyajian-ONeill LA,
Franks RR, Indelicato PA. Concussion (mild traumatic brain injury) and the team physician: a consensus
statement--2011 update. Med Sci Sports Exerc. 2011 Dec;43(12):2412-22. PubMed PMID: 22089299.
McCrory P, Johnston K, Meeuwisse W, Aubry M, Cantu R, Dvorak J, Graf-Baumann T, Kelly J, Lovell M,
Schamasch P. Summary and agreement statement of the 2nd International Conference on Concussion in
Sport, Prague 2004. Br J Sports Med. 2005 Apr;39(4):196-204. Review. PubMed PMID: 15793085.

END OF SERIES

Question 99
A 24-year-old collegiate pitcher has had increasing pain over his medial elbow for 3 months. He has point
tenderness over his medial epicondyle and reproduction of his symptoms with a valgus stress test. What
phase of the throwing cycle most likely will reproduce his symptoms?

1. Early cocking
2. Late cocking
3. Acceleration
4. Deceleration

PREFERRED RESPONSE: 2

DISCUSSION
This patient is experiencing soreness over his medial (ulnar) collateral ligament. Valgus overload is likely
to reproduce his symptoms and is most pronounced during the late cocking phase of the throwing cycle.
In wind up, very little elbow torque is required. In early cocking, the arm is getting loaded and maximum
valgus is not yet achieved at the elbow. In acceleration and deceleration, more force is generated at the
level of the shoulder joint.

RECOMMENDED READINGS
Safran M, Ahmad CS, Elattrache NS. Ulnar collateral ligament of the elbow. Arthroscopy. 2005
Nov;21(11):1381-95. Review. PubMed PMID: 16325092.
Safran MR. Ulnar collateral ligament injury in the overhead athlete: diagnosis and treatment. Clin Sports
Med. 2004 Oct;23(4):643-63, x. Review. PubMed PMID: 15474227.

2013 American Academy of Orthopaedic Surgeons 2013 Sports Medicine Self-Assessment Examination
2013 Sports Medicine Self-Assessment Examination Answer Book 85

Figure 100

Question 100
Figure 100 is the MRI scan of a 52-year-old runner who has right knee pain that has been occurring 10
minutes into her run for 2 months. On examination, she has tenderness over the lateral epicondyle. Her
Ober test result is positive. What is the most appropriate initial treatment?

1. Iliotibial band bursectomy


2. Z-lengthening of the iliotibial band
3. Partial excision of the iliotibial band
4. Home stretching program and cross training

PREFERRED RESPONSE: 4

DISCUSSION
Iliotibial band syndrome (ITBS) is a common cause of lateral knee pain in runners. Potential etiologies
for the pain include repetitive friction, compression, and bursal inflammation. An Ober test is used
to assess iliotibial band tightness. With the patient lying on the unaffected side, the affected leg is
abducted and extended. The test result is positive if the examiner is unable to adduct the leg from this
position. An MRI scan can be helpful in making the diagnosis, but a negative MRI scan does not rule
out ITBS. Studies have reported increased signal intensity on T2-weighted images deep to the iliotibial
band adjacent to the lateral epicondyle, with thickening of the iliotibial band. Nonsurgical treatment is
most appropriate initially and involves activity modification, ice, anti-inflammatory medications, and
stretching. Corticosteroid injection to the iliotibial bursa is also an option to treat acute pain. After the
initial inflammation improves, a strengthening program is started. Multiple surgical procedures have
been described for recalcitrant cases, including iliotibial band excision, Z-lengthening, and iliotibial band
bursectomy.

RECOMMENDED READINGS
Strauss EJ, Kim S, Calcei JG, Park D. Iliotibial band syndrome: evaluation and management. J Am Acad
Orthop Surg. 2011 Dec;19(12):728-36. Review. PubMed PMID: 22134205.
Hariri S, Savidge ET, Reinold MM, Zachazewski J, Gill TJ. Treatment of recalcitrant iliotibial band
friction syndrome with open iliotibial band bursectomy: indications, technique, and clinical outcomes. Am
J Sports Med. 2009 Jul;37(7):1417-24. Epub 2009 Mar 13. PubMed PMID: 19286912.

2013 American Academy of Orthopaedic Surgeons 2013 Sports Medicine Self-Assessment Examination
51% of AAOS Fellows
have unclaimed CME!

Your AAOS membership,


delivering exceptional value to you.

Das könnte Ihnen auch gefallen